Download as pdf or txt
Download as pdf or txt
You are on page 1of 109

CHARTERED ACCOUNTANCY PROFESSIONAL CAP-II

SUGGESTED ANSWER
December 2017

Gf]kfn rf6{8{ PsfpG6]G6\; ;+:yf


The Institute of Chartered Accountants of Nepal
Advanced Accounting
Maximum Marks - 100
Total No. of Questions - 6 Total No. of Printed Pages - 5
Time Allowed - 3 Hours
Marks
Attempt all questions. Working notes should form part of the answer.
1. Batliboi & Co. Ltd. carried on manufacturing business. Its products were sold to
wholesalers and the company had its own retail shop. Adhikary & Co. (P) Ltd.
carried on similar manufacturing business, but all goods produced were sold
through the company's own retail shops.
The summarized balance sheets of the two companies as at 31st March, 2014
were as follows:
Batliboi & Adhikary & Batliboi & Adhikary &
Co. Ltd. Co. (P) Ltd. Co. Ltd. Co. (P) Ltd
(Rs.) (Rs.) (Rs.) (Rs.)
Share Capital Fixed Assets:
Authorized equity Freehold Pro-
shares of Rs. 10 40,00,000 6,00,000 perties at cost 10,00,000 2,50,000
Plant & Machinery
Issued & fully at cost less
paid up 25,00,000 6,00,000 depreciation 13,00,000 1,00,000
23,00,000 3,50,000
P & L A/c 3,40,000 90,000 Current Assets:
Creditors 4,20,000 70,000 Stock 4,80,000 1,20,000
Debtors 2,30,000 80,000
Bank 2,50,000 2,10,000
32,60,000 7,60,000 32,60,000 7,60,000
The Original cost of Plant and Machinery was:
Batliboi & Co. Ltd. Rs. 26,00,000
Adhikary & Co. (P) Ltd. Rs. 2,00,000
The following arrangements were made and carried out on April 1, 2014.
a) Batliboi & Co. Ltd. purchased from the shareholders of Adhikary & Co. (P) Ltd. all
the issued shares @ Rs. 14 per share.
b) The shareholders of Adhikary & Co. (P) Ltd. took over one of the freehold properties
of Adhikary & Co. (P) Ltd. for Rs. 60,000, at the book value of the same. It was
agreed that the amount should be set off against the amount due to them under (a)
above and the balance due to them to be satisfied by the issue of an appropriate
number of equity shares in Batliboi & Co. Ltd. at Rs. 19.50 per share.
The necessary transfer in regard to the setting off the price of the property taken over
by the shareholders against the amount due to them from Batliboi & Co. Ltd. were
made in the books of the two companies.
c) All manufacturing was to be carried on by Batliboi and Co. Ltd. and all retail business
is to be carried on by Adhikary & Co. (P) Ltd. in this connection.
i) Batliboi & Co. Ltd. purchased the whole of Adhikary & Co. (P) Ltd.'s plant and
machinery for Rs. 1,50,000 and certain of their free-hold property (cost Rs.
1,00,000) at Rs. 1,20,000.
ii) Adhikary & Co. (P) Ltd. purchased Batliboi & Co. Ltd.‘s freehold retail shop
Buildings (Cost to Batliboi & Co. Ltd., Rs. 75,000) at Rs. 60,000 and took over
the retail stock at Rs. 80,000 at the book value.
d) Batliboi & Co. Ltd. drew a cheque in favour of Adhikary & Co. (P) Ltd. for
the net amount due, taking into account all the matters mentioned above.
e) Immediately after the transfer of shares in (a) above. Adhikary & Co. (P)
Ltd. declared and paid dividend of Rs. 60,000 (ignore income-tax).
You are required to prepare the Balance Sheets of Batliboi & Co. Ltd. and
Adhikary & Co. (P) Ltd. immediately after the completion of the above
transaction. 20
ANSWERS:
Balance Sheet of Batliboi & Co. as on 1st April, 2014

Liabilities Rs. Assets Rs.


Share Capital Fixed Assets
Authorized Freehold properties
4, 00,000 shares of Rs. 10 as per last Balance
Each 40, 00,000 Sheet 10, 00,000
Issued, subscribed & paid-up Addition during the
2, 90,000 shares of Rs. 10 each Year 1, 20,000
Fully paid of which 40,000 1120000
Shares were issued pursuant Less: sold during the 75,000
To contract without pay- Year 1045000
Ment being received in cash 29, 00,000 Plant and Machinery:
As per 26, 00,000
Balance Sheet
Reserved & Surplus Addition during 1, 50,000
The year
Share premium A/c 3, 80,000 27, 50,000
Profit & Loss A/c 3, 25,000 Less: Depreciation 13, 00,000
Current Liabilities & 14, 50,000
Provisions: Investments
Sundry Creditors 4, 20,000 Shares in subsidiary 7, 80,000
Company
Current, Assets, Loans & Advances
Stock in trade 4, 00,000
Sundry debtors 2, 30,000
Cash at Bank 1, 20,000
40, 25,000 40, 25,000
Working Notes:
(1) Value of shares in Adhikary & Co. (P) Ltd. Rs.
60,000 Shares @ Rs. 14 per share 8, 40,000
Less: Value of freehold property 60,000
Net amount due 7, 80,000

No of shares issued 7, 80,000/ Rs. 19.5= 40,000


Amount credited to Share Capital 4, 00,000
Amount credited to Share premium 3, 80,000
7, 80,000
(2) Rs. 15,000 loss on the sale of Building to Adhikary & Co. (P) Ltd. has been debited to
the profit & Loss Amount.
(3) Investment A/c has been credited by dividend received Rs. 60,000 out of pre-
acquisition profit.(780,000+60,000-60,000)
(4) Amount payable Adhikary & Co. (P) Ltd. Rs.
Value of assets purchased from Adhikary & Co. (P) Ltd. 2, 70,000
Less: Value of assets sold 1, 40,000
1, 30,000
Value of freehold property taken over by shareholders
Of Adhikary & Co. (P) Ltd. 60,000
1, 90,000
(5) Cash Balance:
As given 2, 50,000
Add: Dividend received 60,000
3, 10,000
Less: paid to Adhikary & Co. Ltd. including
Rs. 60,000 for assets taken over by its erstwhile shareholders 1, 90,000
1, 20,000

Balance Sheet of Adhikary & Co. (P) Ltd. as on 1st April, 2014

Liabilities Rs. Assets Rs.


Share capital Fixed Assets
Authorized Freehold properties:
60,000 shares of Rs. 10 each 6, 00,000 as per last Balance
Issued, subscribed & paid up Sheet 2, 50,000
60,000 shares of Rs. 10 Addition during the year 60,000
Each full paid 6, 00,000 3, 10,000
Reserved and Surplus Less: Sold during the year 1, 60,000 1,50,000
Plant & Machinery
Profit & Loss Account 1, 00,000 Cost 2, 00,000
Current Liabilities & Provision Less: Sold (Cost) 2, 00,000 nil
Sundry Creditors 70,000 Current Assets, Loan & Advances
Stock in trade 2,00,000
Sundry debtors 80,000
Cash at Bank 3,40,000
7, 70,000 7, 70,000
Working Notes: Rs.
1) Profit & Loss Account (given) 90,000
Add: Profit on sale of machinery and freehold property 70,000
1, 60,000
Less: Dividend paid 60,000
1, 00,000

2) Freehold properties have been reduced by Rs. 1, 00,000 transferred to Batilboi &
Co. & Rs. 60,000 taken over by the shareholders of Adhikari & Co. (P) Ltd.
3) Cash at Bank: Balance as given Rs.
Add: Received from Batliboi & Co. 2, 10,000
1, 90,000
4, 00,000
Less: Dividend Paid 60,000
3, 40,000
2.
a) Eddi and Freddi carrying on business in partnership sharing Profit and
Losses equally, wished to dissolve the firm and sell the business to Sreddi
Ltd. on 31.03.2074, when the firm‘s position was as follows:
Balance Sheet as at 31.03.2074
Liabilities Amount (Rs.) Assets Amount (Rs.)
Eddi‘s Capital 175,000 Land and Building 125,000
Freddi‘s Capital 130,000 Furniture 52,500
Sundry Creditors 80,000 Stock 112,500
Debtors 91,000
Cash 4,000
Total 385,000 Total 385,000
The arrangement with Sreddi Limited Company was as follows:
i) Land and Building was purchased at 20% more than the book value.
ii) Furniture and stock were purchased at book values less 15%.
iii) The goodwill of the firm was valued at Rs. 40,000.
iv) The firm‘s debtors, cash and creditors were not to be taken over, but the
company agreed to collect the book debts of the firm and discharge the
creditors of the firm as an agent, for which services, the company was
to be paid 5% on all collections from the firm‘s debtors and 3% on cash
paid to firm‘s creditors.
v) The purchase price was to be discharged by the company in fully paid
equity shares of Rs. 10 each at a premium of Rs. 2 per share.
The company collected all the amounts from debtors. The creditors were
paid off less by Rs. 1,000 allowed by them as discount. The company paid
the balance due to the vendors in cash.
Prepare the Realization Account, the Capital Accounts of the partners
and the Cash Account in the books of partnership firm. 10
b) Following information as at second quarter ending 2073 were drawn from
the records of Minamma Bank Limited as under:
Loan outstanding for Amount (Rs.)
Up to 3 months 1,673,000
More than 3 months but not more than 6 months 13,612
More than 6 months but not more than 12 months 782
More than 12 months 2,198
Total 1,689,592
The bank has not restructured or rescheduled any of its credit.
Following additional information relating to previous quarter ending were
extracted from the records of the bank:
Particulars Amount (Rs.)
Paid up equity share capital 171,010
Statutory general reserve 155,432
Retained earnings 87,886
General loan loss provision 16,983
Exchange equalization reserve 22,313
Un-audited current year cumulative profit 31,991
Deferred revenue expenses 2,884
The bank is in the process of preparing the documents for quarterly
reporting. The bank has also provided a term loan of Rs. 125,000 to a single
party during the period under review. As a reporting and compliance officer
of the bank you are required to calculate movement in loan loss provision
amount. 10
ANSWERS:
a)
Amount in NPR
Dr Realization A/c Cr
Particulars Amount Particulars Amount
To Land & Building 125,000 By Sundry Creditors 80,000
To Furniture 52,500 By Sreddi Ltd Co- 330,250
Purchase consideration (W.N-1)
To Stock 112,500 By Sreddi Ltd Co- Sundry
Debtors 91,000
To Debtors 91,000 Less- Commission 86,450
(5% on 91,000) 4,550
To Sreddi Ltd Co- 79,000
Sundry Creditors
To Sreddi Ltd Co- 2,370
Commission
(3% on 79,000)
To Profit transferred to 34,330
Eddi‘s Capital 17,165
Freddi‘s Capital 17,165
Total 496,700 Total 496,700

Amount in NPR
Dr Capital Account Cr
Particulars Eddi Freddi Particulars Eddi Freddi
To Shares in 187,023 143,227 By Balance b/d 175,000 130,000
Sreddi Ltd (W.N-
2)
To Cash –Final 5,142 3,938 By Realization A/c 17,165 17,165
Payment
Total 192,165 147,165 Total 192,165 147,165

Amount in NPR
Dr Cash Account Cr
Particulars Amount Particulars Amount
To Balance b/d 4,000 By Eddi‘s Capital – Final 5,142
payment
To Sreddi Ltd Co- 5,080 By Freddi‘s Capital – Final 3,938
(Amount realized from Debtors payment
less amount paid to creditors)
( W.N-3)
Total 9,080 Total 9,080

Working Notes
1. Computation of Purchase Consideration

Particulars Amount
Land & Building (125,000+20% of 125,000) 150,000
Furniture (52,500-15% of 52,500) 44,625
Stock (112,500-15 % of 112,500) 95,625
Goodwill ( given) 40,000
Total 330,250
2. The shares received from the company have been distributed between the two partners
Eddi & Freddi in the ratio of their final claims i.e., 192,165: 147,165

No of shares received from the company=


Eddi gets ( Shares valued at 15,585*12 =NPR.187, 023.
Freddi gets the remaining 11,936 shares valued at 11,936*12 = NPR.143, 227
3. Calculation of net amount received from Sreddi Ltd on account of amount realized from
debtors less amount paid to creditors.
Particulars Amount
Amount realized from debtors 91,000
Less: Commission for realization from debtors (5% on 91,000) 4,550
86,450
Less: Amount paid to creditors 79,000
7,450
Less: Commission for cash paid to creditors (3% on 79,000) 2,370
Total 5,080
b) As per the provision of the Directive 3 of unified Directives, a bank can provide a credit
up to 25% of its core capital to a single party. This limit is called the single obligatory
limit (SOL). While calculating the SOL, core capital of previous quarter shall be taken as
base. In case any excess credit than SOL has been provided by the bank, additional 100%
provision shall be made for such excess credit amount.
Before calculating the provision amount, SOL of the bank shall be tested upon.
Computation of SOL and credit amount in excess of SOL
Particulars Amount
Core Capital
Paid up Equity Share Capital 171,010
Statutory General reserve 155,432
Retained earnings 87,886
Un-audited current year cumulative profit 31,991
Less: Deferred Revenue expenses (2,884)
Total Core capital 443,435
Single obligatory limit ( 25% of the core capital) 110,859
Loan to single party 125,000
Loan in excess of SOL 14,141
Computation of loan loss provision amount as per provisions of the Directive 2 of the NRB
Computation of Loan Loss Provision amount
Categories Loan Amt Provision Provision
Rate Amount
Not due or <=3 months Pass 1,673,000 1% 16,730
>3 months <= 6 mths Sub-standard 13,612 25% 3,403
>6 months <= 12 mths Doubtful 782 50% 391
>12 months Loss 2,198 100% 2,198
Total 1,689,592 22,722
Additional provision for loan in excess of SOL 14,141
Total Provision amount 36,863

Minamma Bank Ltd


Movement in Provision Amount
For Second Quarter of Fiscal Year 2073/74
Amount in NPR
Particulars Amount
Opening Provision amount 16,983
Closing Provision amount 36,863
Movement in provision amount (addition during the quarter) 19,880

3.
a) On 19th Mansir, 2073 premises of National Trading were destroyed by fire,
but sufficient records were saved, wherefrom the following particulars were
ascertained:
Rs.
Stock at cost on 1.4.2072 36,750
Stock at cost on 31.03.2073 39,800
Purchases less returns during 2072/73 1,99,000
Sales less return during 2072/73 2,43,500
Purchases less returns during 1.4.2073 to 19.8.2073 81,000
Sales less returns during 1.4.2073 to 19.8.2073 1,15,600
National Trading closes his books every 31st Ashadh. In valuing the stock
for the balance sheet as at 31st Ashadh 2073 Rs. 1,150 had been written off
on certain stock which was a poor selling line having the cost Rs. 3,450. A
portion of these goods were sold in Ashoj, 2073 at a loss of Rs. 125 on
original cost of Rs. 1,725. The remainder of this stock was now estimated to
be worth the original cost. Subject to the above exceptions, gross profit has
remained at uniform rate throughout. The stock salvaged was Rs. 2,900.
Show the amount of the claim of stock destroyed by fire. Memorandum
Trading Account to be prepared for the period from 1-4-2073 to 19-8-2073
for normal and abnormal items. 10
b) Mahakali Bank Ltd. issues shares of Rs. 10 each at a premium of 10%
payable as follows:
On application Rs. 2
On allotment Rs. 3 (including premium)
On first call Rs. 2
On final call Rs. 4
Mahesh who was holding 50 shares did not pay his allotment and first call
and his shares were forfeited. Suresh, who was holding 30 shares, did not
pay first call and his shares also were forfeited. Journalize transaction
relating to forfeiture of shares. 5
ANSWERS:
a)
National Trading
Trading Account for the year ended on 31st Ashadh, 2073
Rs Rs Rs
To Opening Stock 36,750 By Sales A/c 2,43,500
To Purchases 1,99,000 By Closing Stock
To Gross Profit 48,700 As valued 39,800
Add: Amount written off 1,150 40,950
to restore stock to full
cost
2,84,450 2,84,450

The normal rate of gross profit to sales is =48,700 x 100 =20%


2, 43,500
Memorandum Trading Account up to 19, Mansir, 2073
Normal Abnormal Total Normal Abnormal Total
Items Items Items Items
Rs Rs Rs Rs Rs Rs
To 37,500 3,450* 40,950 By Sales 1,14,000 1,600 1,15,600
Opening
Stock
To 81,000 - 81,000 By Loss - 125 125
Purchases
To Gross 22,800 - 22,800 By 27,300 1,725 29,025
Profit Closing
(20% on Stock
Rs (bal.fig)
1,14,000)
1,41,300 3,450 1,44,750 1,41,300 3,450 1,44,750
*at cost.
Calculation of Insurance Claim
Particulars Rs.
th
Value of Stock on 19 Mansir, 2073 29,025
Less :Salvage (2,900)
Loss of Stock 26,125
Therefore insurance claim will be for Rs. 26, 125 only.

b)
Share capital A/C –Dr 50*6 300
Securities Premium A/c –Dr (50*1) 50
To forfeited shares A/C (50*2) 100
To share allotment A/C (50*3) 150
To Share first call A/C( 50*2) 100
( Being forfeiture of Mahesh Shares)
Share Capital A/C 30*6 180
To forfeited shares A/C (30*4) 120
To share first call A/C(30*2) 60
(Being forfeiture of Suresh A/C)
Note: in case of Mahesh securities premium account has been cancelled by debiting it
because the premium has not been paid but in case of Suresh it has not been done
because the premium has been paid, premium once received cannot be cancelled.

4.
a) From the following information of Reliable Marine Insurance Ltd. for the
st
year ending 31 Ashadh, 2074 find out the 10
i) Net premiums earned
ii) Net claims incurred
Particulars Direct Business (Rs.) Re-insurance (Rs.)
Premium:
Received 88,00,000 7,52,000
Receivable– 01.04.2073 4,39,000 36,000
Receivable– 31.03.2074 3,77,000 32,000
Paid 6,09,000
Payable– 01.04.2073 27,000
Payable– 31.03.2074 18,000
Claims:
Paid 69,00,000 5,54,000
Payable– 01.04.2073 89,000 15,000
Payable– 31.03.2074 95,000 12,000
Received 2,01,000
Receivable– 01.04.2073 40,000
Receivable– 31.03.2074 38,000
b) M/s Baboon & Co. has four departments A, B, C, D. Each department was
being managed by a departmental manager whose commission was 10% of
the respective departmental profit, subject to a minimum of Rs. 6,000 in
each case. Inter departmental transfers took place at a loaded price as
follows.
From Department A to Department B: 10% above cost
From Department C to Department D: 20% above cost
From Department A to Department D: 20% above cost
From Department C to Department B: 20% above cost
For the year ended 31st Ashadh, 2074 the company had already prepared and
closed the Departmental Trading and Profit and Loss Account. Subsequently
it was discovered that the closing stock of departments had included inter
departmentally transferred good at loaded price instead of cost price.
From the following information, prepare a statement re-computing the
departmental profit or loss. 5

(Rs.)
Particulars Dept. A Dept. B Dept. C Dept. D
Final Profit/ Loss 38,000 50,400 72,000 1,08,000
(Loss) (Profit) (Profit) (Profit)
Inter departmental 70,000 4,800
transfers included at (22,000 from Dept. (3,600 from Dept.
loaded price in the A and 48,000 from C and 1,200 from
departmental stock Dept. C) Dept. A)
ANSWERS:
a)
Net Premium earned
Particulars Rs. Rs.
Premium from direct business received 88,00,000

Add: Premium on re-insurance accepted 7,52,000


Add :Receivable as on 31.03.2074 32,000
Less: Receivable as on 01.04.2073 (36,000) 7,48,000
95,48,000
Less: Premium on re-insurance ceded 6,09,000
Add: Payable ason31.03.2074 18,000
Less :Payable as on 01.04.2073 (27,000) (6,00,000)
Net Premium earned 89,48,000

(ii) Net Claims incurred


Rs.
Claims paid on direct business 69,00,000
Add: Re-insurance 5,54,000
Add: Outstanding as on 31.3.2073 12,000
Less: Outstanding as on 1.4.2074 (15,000) 5,51,000
74,51,000
Less: Claims received from re-insurance 2,01,000
Add: Outstanding as on 31.3.2074 38,000
Less: Outstanding as on 1.4.2073 (40,000) (1,99,000)
72,52,000
Add: Outstanding direct claims at the end of the year 95,000
73,47,000
Less :Outstanding claims at the beginning of the year (89,000)
Net claims incurred 72,58,000
b)
Statement of re-computation of department profit/Loss
Particulars Dept. A Dept. B Dept. C Dept. D
Final profit/Loss as computed earlier (38,000) 50,400 72,000 1,08,000
Add –Department manager‘s commission 6,000 6,000 8,000 12,000
already deducted from profit ( W.N-1)
Profit before charging manager‘s (32,000) 56,400 80,000 120,000
commission
Loss- Profit earned due to transfer of (2,200) - (8,600) -
goods at loaded price and included in
departmental unsold stock ( WN-2)
Correct departmental profit before (34,200) 56,400 71,400 120,000
charging manager‘s commission
Manager‘s Commission @10% of (6,000) (6,000) (7,140) (12,000)
departmental profit subject to a minimum
of Rs.6,000
Adjusted Profit/Loss (40,200) 50,400 64,260 108,000

Working Notes
Manager‘s commission is payable @10 % of departmental profit before charging such
commission (subject to a minimum of Rs.6, 000). Hence manager‘s commission already
deducted will be as follows.
Departments Profit /Loss after Manager’s Commission
charging manager’s
commission
A (38,000) 6,000
B 50,400 1/9 of Rs.50,400 or 6,000 whichever is higher
i.e. 6,000
C 72,000 1/9 of Rs.72,000=8,000
D 108,000 1/9 of Rs 1,08,000=12,000

2. Unrealized profit on unsold departmental stock:


Profit earned by Department A by transfer of stock Amount
Department B@110%= Rs.22,000*10/110=2,000
Department D @120%= Rs.1,200*20/120=200 2,200

Profit earned by the Department C by transferring stock to


Department D@120%= Rs.3,600*20/120=600
Department B@120%= Rs.48,000*20/120=8,000 8,600

5.
a) From the following information, find out the risk weighted exposure for
operational risk assuming 11% is the capital adequacy requirement. 5
(
Rs.)
Particulars Year I Year II Year III
Interest Income (Rs.)
1,466,454 1,626,474 1,775,583
Interest Expenses 561,964 648,842 767,411
Commission and Discount Income 132,816 165,448 193,224
Other Operating Income 137,301 198,130 151,637
Exchange Fluctuation Income 41,301 52,325 40,329
Addition in interest Suspense Account 25,693 34,376 36,711
Non-Operating and Extraordinary Income 2,795 1,887 100,257
b) Nepa Roadways has taken a transit insurance policy. Suddenly in the year
2073-2074 the percentage of accident has gone up to 12% and the company
wants to recognize insurance claim as revenue in 2073-2074 in accordance
with relevant Accounting Standard. Do you agree? 5
c) Kalika Constructions undertook the construction of a highway on
01.04.2073. The contract was to be completed in 2 years. The contract price
was estimated at Rs. 150 crores. Up to 31.03.2074 the company incurred Rs.
120 crores on the construction. The engineers involved in the project
estimated that a further Rs. 45 crores would be incurred for completing the
work.
What amount should be charged to revenue for the year 2073-2074 as per
the provision of Nepal Accounting Standard 11 ―Construction Contract"?
Show the extract of the Profit & Loss A/c in the books of Kalika
Constructions. 5

ANSWERS:
a)
Net Interest Income 904,490.00 977,632.00 1,008,172.00

Commission and Discount Income 132,816.00 165,448.00 193,224.00

Other Operating Income 137,301.00 198,130.00 151,637.00

Exchange Fluctuation Income 41,301.00 52,325.00 40,329.00


Addition/Deduction in Interest Suspense during the
period 25,693.00 34,376.00 36,711.00

Gross Income( a) 1,241,601.00 1,427,911.00 1,430,073.00


Alfa (b) 15% 15% 15%

Fixed percentage of Gross Income © 186,240.15 214,186.65 214,510.95


Capital Requirement for operational risk (d )
(average of C) 204,979.25

Risk weight in times(e) 11

Equivalent Risk Weight Exposure (f=d*e) 225,4771.75

Net Interest Income


Year I Year II Year III
Interest Income 1,466,454.00 1,626,474.00 1,775,583.00
Interest Expenses 561,964.00 648,842.00 767,411.00
Net Interest Income 904,490.00 977,632.00 1,008,172.00

b) NAS 18 on ―Revenue‖ defines revenue as ―gross inflow of economic benefits during the
period arising in the course of the ordinary activities of an entity when those inflows result in
increases in equity, other than increases relating to contributions from equity participants‖
To recognize revenue NAS 18 requires that revenue arises from ordinary activities and that it
can be measured reliably and it is probable that the economic benefits associated with the
transaction will flow to the entity. As per the Standard, where the ability to assess the
ultimate collection with reasonable certainty is lacking at the time of raising any claim,
revenue recognition is postponed to the extent of uncertainty involved. In such cases, it may
be appropriate to recognize revenue only when it is reasonably certain that the ultimate
collection will be made.
In the given case, Nepa Roadways wants to recognize Insurance claim because it has
increased over the previous year. But, there are uncertainties involved in the settlement of
the claim. Also, the claim does not seem to be in the course of ordinary activity of the
company.
Hence, Nepa Roadways is not advised to recognize the Insurance claim as revenue.
Marking Scheme
3 marks for provision
2 marks for decision
c)
Statement showing the amount to be charge to Revenue as per NAS 11
Particulars Rs. in Crores
Cost of construction incurred up to 31.03.2074 120
Add: Estimated future Cost 45
Total estimated cost of construction 165
Degree of completion (120/165×100) 72.73%
Revenue recognized (72.73% of 150) 109 ( approx.)
Total foreseeable loss (165-150) 15
Less: Loss for the current year (120-109) 11
Loss to be provided for 4

Profit and Loss Account (Extract)


Particulars Rs. in crores Particulars Rs. in crores
To Construction Costs 120 By Contract Price 109
To Provision for Loss 4 By Net Loss 15
124 124

6. Write short notes on: (5×3=15)


a) Fair Value Accounting
b) Exchange Equalization Fund
c) Underlying assumption on Preparation and presentation of Financial Statement
d) Sale and Lease Back Transactions as per NAS 17.
e) Write any five peculiar features of Farm Accounting.
ANSWERS:
a) Fair Value is the amount for which an asset could be exchanged or a liability settled between
knowledgeable willing parties in an arm's length transaction. Fair value is measured using the
price in the principal market for the asset or liability (i.e. the market with the greatest volume and
level of activity for the asset or liability) or, in the absence of a principal market, the most
advantageous market for the asset or liability. Detailed guidance shall be required for measuring
the fair value of liabilities, including a description of the compensation that market participants
would demand to take on an obligation.
b) Section 45 of the Bank and Financial Institution Act, 2073 requires that every Bank and Financial
Institution which has obtained a license to deal in foreign exchange must maintain a Exchange
Equalization Fund. The Bank and Financial Institution must transfer 25% of the revaluation
profit earned as a result of changes in the exchange rates of foreign currencies other than the
Indian currency every year at the end of the same fiscal year to the exchange equalization fund.
The amount credited to the Exchange Equalization Fund may not be spent or Transferred for any
purpose other than the adjustment of losses resulting from the devaluation of foreign currencies
without the approval of the Nepal Rastra Bank.

c)
Accrual Basis
In order to make their objectives financial statements are prepared on the accrual basis of
accounting. Under this basis, the effects of transaction and other events are recognized when they
occur (and not as cash or equivalent is received or paid) and they are recorded in the accounting
records and reported in financial statements of the periods to which they relate.
Going Concern
The financial statements are normally prepared on the assumption that an entity is a going
concern and will continue in operation for foreseeable future. Hence, it is assumed that the entity
has neither the intention nor the need to liquidate or curtail materially the scale of operations; if
such intention or need exists, the financial statements may have to be prepared on a different
basis.
d) As per NAS 17 on ‗Leases‘, a sale and leaseback transaction involves the sale of an asset by the
vendor and the leasing back of same asset to the vendor. The lease payments and the sale price
are usually interdependent, as they are negotiated as a package. The accounting treatment of a
sale and lease back transaction depends upon the type of lease involved.
If a sale and leaseback transaction results in a finance lease, any excess of sale proceeds over the
carrying amount shall not be immediately recognized as income by a seller-lessee. Instead it shall
be deferred and amortized over the lease term.
If sale and leaseback transaction results in a operating lease, and it is clear that the transaction is
established at fair value, any profit or loss should be recognized immediately. If the sale price is
below fair value any profit or loss should be recognized immediately except that, if the loss is
compensated by future lease payments at below market price, it should be deferred and amortized
in proportion to the lease payments over the period for which the asset is expected to be used. If
the sale price is above fair value, the excess over fair value should be deferred and amortized
over the period for which the asset is expected to be used.
Poultry, livestock and other agricultural produces.
e) While preparing the farm account one should be conversant with the peculiar features of farm
accounting which are as below:
1. The business is family type and there may be single bank account both for business and
family purpose.
2. A large chunk of the farm produce is appropriated towards consumption by the family
members.
3. The family provides labor for the farm in addition to the time devoted for management.
4. There are several divisions in the farm such as crops, dairy, poultry etc.
5. The output of certain division becomes the input of other divisions.
6. Farming operations are fraught with uncertainties as to the weather, pests, market price of the
input and outputs and government policies.
7. In some cases, agriculture is a part time occupation.
8. Due to the intrinsic nature, there are difficulties in ascertaining the value of standing crops
poultry, livestock and other agricultural produces.
Audit and Assurance
Maximum Marks - 100

Total No. of Questions- 7 Total No. of Printed Pages- 9

Time Allowed - 3 Hours


Marks
Attempt all questions.
1. As an auditor, give your opinion with explanations on the following cases: (45=20)
a) You are an engagement manager for an audit team for M/s Sun Life Ltd. for
financial statements as on Ashadh 31, 2073. Your team is about to drafting
management letter after all transaction testing and review; the board of director of
M/s Sun Life Ltd. has appointed new CEO. After assuming his office, he decided to
change few general managers and auditor for FY 2072-73 with immediate effect.
b) M/s Raddison Hotel on 15.4.2073 imported two Mercedes Benz from Germany at a
price Euro 200 thousand each upon terms of credit that price should be settled
within three months from the date of purchase. The company capitalized the asset
and created a liability for the capital goods converting the foreign currency liability
to Nepalese Rupees at a rate of exchange prevailing as on 15.4.2073. When the
company settled the liability on 30.8.2073, it had to incur an additional amount of
Rs.15,00,000 due to foreign exchange rate on the date of settlement. It added this
additional amount of exchange variation in the capital cost of the asset and charged
depreciation upon an enhanced amount of asset value from 30.8.2073.
c) Mr. A Kumar and Mr. B Kumar are the two chartered accountants just qualified
their exams and took membership from ICAN in Bhadra 2074. A Kumar, who
without holding the Certificate of Practice, signed a document in capacity of the
member holding Certificate of Practice and B Kumar, as being a member of ICAN,
committed an act contrary to the provisions of Section 41of ICAN Act.
d) Mr. Rajaram is a director in M/s P Ltd. and also in M/s Q Ltd. M/s Q Ltd. purchases
goods from M/s P Ltd. The two companies reported the transaction as transaction
with related party as on Ashadh 31, 2074 under NAS-24.

Answer:
a) As per provision contained in Section 119 of companies Act 2063 relating to removal of
appointed auditor, no auditor appointed pursuant to Companies Act shall be removed pending
the completion of audit of accounts of any financial year for which he/she was appointed as the
auditor.

However, if any auditor breaches the code of conduct of auditors or does any act against the
interest of the company which has appointed him as the auditor or commits any act contrary to
the prevailing law, such auditor may be removed through the same process whereby he/she was
appointed as auditor, by giving prior information to the Nepal Chartered Accountants Institute,
and with the approval of the regulatory authority, if any authorized by the prevailing law for the
regulation of business of the company concerned , and failing such authority, with the approval
of the Office of company registrar.
In case of M/s Sun Life Ltd, CEO is not the person to appoint and remove statutory auditor
and the actions taken by the CEO is null and void and additionally, an appropriate
communication to those charged with governance is required.

b) Nepal Accounting Standards (NAS) 21 states that the exchange differences on the settlement of
monetary items at a rate different from those at which they were translated on initial
recognition during the period or in previous financial statements shall be recognized in profit or
loss in the period in which they arise. Therefore, in view of the above, the M/s Raddison Hotel
should charge the additional amount of Rs.15,00,000 to profit and loss account in accordance
with the said standard and not to capitalize.

c) Section 41 of Nepal Chartered Accountants Act, 2053 has made different level of punishment
for different levels of culpability. Here in case of A Kumar, If a person, who has not obtained a
Certificate of Practice and is proved to have signed any document in capacity of the member
holding Certificate of Practice, shall be liable to punishment with a penalty up to two thousand
rupees or imprisonment for a period of up to three months or both.

In case of B Kumar, if a member, who commits any act contrary to the provisions of this Act or
Regulations framed under this Act other than the provisions of this section, shall be suspended
for a maximum period of five years and shall be liable of punishment with a maximum penalty
of two thousand rupees or imprisonment for a maximum period of three months or both.
So, A Kumar and B Kumar are to be punished accordingly.

d) According to the Provisions of NAS-24, the given transaction between P Ltd. and Q Ltd. is not
a related party transaction. According to the standard, related party relationship includes
enterprises owned by directors or major shareholders of the reporting enterprise and enterprises
that have a member of key management in common with the reporting enterprise. In the given
case, none of the enterprises is owned by Rajaram. He is only a director in both the enterprises
not a Key Management Personnel (Such as Managing Director, Whole time Director, etc).
Therefore, P Ltd should not report the transaction as related party transaction.

2. Give your comments on the following cases: (45=20)


a) A proprietary audit firm of Fellow Chartered Accountants has accepted the
engagement to audit the accounts of a company with annual turnover of Rs. 10
million at the audit fee of Rs. 12,000. The firm anticipates that the audit will
consume estimated time of 3 man days of the Chartered Accountant.
b) During the course of audit of a company for fiscal year 2073/74, the finance director
of the company offered your audit team a free weekend at Fulbari Resort Pokhara.
c) The auditor of H Ltd. wanted to obtain confirmation from its creditors. But the
management made a request to the auditor not to seek confirmation from certain
creditors citing disputes. Can the auditor of H Ltd. accept to this request?
d) M/s KBC Associates, a Chartered Accountant was engaged by Hanuman& Co.
Ltd. for auditing their accounts. He sent his letter of engagement to the Board of
Directors, which was accepted by the company. In the course of audit ofthe
company, the auditor was unable to obtain appropriate sufficient audit evidence
regarding receivables. The client requested for a change in the terms of engagement.
Offer your comments in this regard for acceptance of changes in terms of
engagement.

Answer:
a) As per the decision of the council, a FCA member holding COP shall charge the audit fee to his
clients and the fee shall not be less than Rs. 15,000. In the present case, the FCA member has
charged Rs. 12,000 to a private company is less than the minimum fee. The auditor should have
charged at least Rs. 15,000 as the audit fee. Hence the member seems to have not followed the
directives of the council and accordingly may be subject to disciplinary action.

b) As per the Section 260 of code of ethics issued by ICAN, accepting gifts or hospitality from
an audit client may create self-interest threat and familiarity threats. If a firm or member of
audit team accepts gifts or hospitality unless the value is nominal and inconsequential the threat
created would be so significant that no safeguard could reduce the threats to an acceptable low
level. Consequently a firm or member of the audit team shall not accept such gifts or
hospitality.

The given case represents a self-interest threat as the acceptance of goods and services unless
value of such goods or service offered is very insignificant .As it is unlikely that a weekend at a
luxury hotel for whole audit team has an insignificant value, then, this offer should politely be
declined.

c) NSA 505, ―External Confirmations‖, establishes standards on the auditor‘s use of external
confirmation as a means of obtaining audit evidence. It requires that the auditor should employ
external confirmation procedures in consultation with the management.

The auditor may come across certain situations in which the management may request him not
to seek external confirmation from certain parties because of some reasons, for example, due to
a dispute with the particular creditor or debtor.

If the management refuses to allow the auditor to a send a confirmation request, the auditor shall
 Inquire as to Management‘s reasons for the refusal, and seek audit evidence as to their
validity and reasonableness,
 Evaluate the implications of management‘s refusal on the auditor‘s assessment of the
relevant risks of material misstatement, including the risk of fraud, and on the nature,
timing and extent of other audit procedures, and
 Perform alternative audit procedures designed to obtain relevant and reliable
audit evidence.

If the auditor concludes that management‘s refusal to allow the auditor to send a confirmation
request is unreasonable or the auditor is unable to obtain relevant and reliable audit evidence
from alternative audit procedures, the auditor shall communicate with Those Charged With
Governance (TCWG) and also determine its implication for the audit and his opinion.
d) NSA 210 ―Agreeing the Terms of Audit Engagement‖ deals with the auditor‘s responsibilities
in agreeing the terms of the audit engagement with management. As per NSA 210, if prior to
completing the audit engagement, the auditor is requested to change the audit engagement to an
engagement that conveys a lower level of assurance, the auditor shall determine whether there
is reasonable justification for doing so.

The auditor shall not agree to a change in the terms of the audit engagement where there is no
reasonable justification for doing so. If the terms of the audit engagement are changed, the
auditor and management shall agree on and record the new terms of the engagement in an
engagement letter or other suitable form of written agreement.

If the auditor is unable to agree to a change of the terms of the audit engagement and is not
permitted by management to continue the original audit engagement, the auditor shall:

 Withdraw from the audit engagement where possible under applicable law or regulation;
and
 Determine whether there is any obligation, either contractual or otherwise, to report the
circumstances to other parties, such as Those Charged With Governance (TCWG), owners
or regulators.

3. Answer the following: (35=15)


a) What are the general limitations of audit?
b) Pandit & Associates is the firm of practicing chartered accountants. The firm accept
the audit of Siddhartha Ltd. where Government of Nepal (GoN) holds 60 %
ownership. What is the responsibility of Pandit & Associates before accepting such
audit.
c) Write down examples of situations where external confirmations may be used by the
auditor.
Answer:
a) An audit in accordance with NSA is designed to provide reasonable assurance, so a degree of
imprecision is inevitable due to some inherent limitations as mentioned below:

i. Test Checking/Sampling:- Auditor uses sampling during performance of audit. Audit based
on test nature does not detect all frauds and errors.

ii. Nature of evidence:- The evidence obtained by the auditor are persuasive rather than
conclusive. They only enable auditor to provide reasonable assurance (not absolute
conclusion).

iii. Judgement: - Auditor uses professional judgement while obtaining audit evidence and
evaluating the reasonableness of assertion and estimates made by the management. The
judgement made by the auditor may not always be correct.

iv. Inherent limitation of internal control: - Internal controls suffer from limitation such as
collusion among the employees or abuse of authority by management.
vi. Constraint of Time:- Since most of the audits are carried out only after the year end and
have similar deadlines. Provided limited time period auditor may not able to observe all the
transactions resulting possibilities of oversight.

b) As per the provision of section 7 (2) Audit Act 2048, the auditor of companies where GoN
holds majorities of share should be done in consultation with the Office of Auditor General of
Nepal. Further the practicing chartered accountant as per section 34 (13) of ICAN Act 2053
should not accept his appointment as an auditor of an organization without ascertaining that all
required procedures for appointment as the auditor under the prevailing law has been duly
fulfilled.

In the light of aforesaid provisions of Audit Act 2048, Pandit & Associates before accepting
appointment, should ensure that whether Siddhartha Ltd. has consulted with the Office of
Auditor General of Nepal for their appointment.

c) As per provision sated in NSA 505 (External Confirmations); external confirmations are
frequently used in relation to account balances and their components, but need not be restricted
to these items. Accordingly examples of situations where external confirmations may be used
by the auditor are:

i. bank balances and other information from bankers,


ii. accounts receivable balances,
iii. stocks held by third parties at bonded warehouses for processing or on consignment,
iv. property title deeds held by lawyers or financiers for safe custody or as security,
v. investments purchased from stockbrokers but not delivered at the balance sheet date,
vi. loans from lenders, and
vii. accounts payable balances.

4. Answer/Comment on the following: (35=15)


a) Explain the provision related to Second Opinions with reference to Code of Ethics.
b) Explain the various types of threat explained in ICAN Code of Ethics along with
examples.
c) Pradhan Ghimire & Associates, a firm of chartered accountant has two partners. The
firm is already holding audit of 100 companies including audit of 20 public limited
companies. The firm is further offered for the audit of Premier Investment Limited.
Answer:
a) Section 230 of Code of Ethics contains the provision relating to Second Opinions.
When asked to provide second opinion on the application of accounting, auditing, reporting or
other standards or principles to specific circumstances or transactions by a company or on
behalf of a company or any entity that is not an existing client, a professional accountant in
public practice shall evaluate the significance of any threats and apply safeguards where
necessary to eliminate them or reduce them to an acceptable level.

The existence and significance of the threat will depend on the circumstances of the request and
all the other available facts and assumptions relevant to the expression of professional
judgment. Examples of such safeguards include seeking client permission to contact the
existing accountant, describing the limitations surrounding any opinion on communications
with the client and providing the existing accountant with a copy of the opinion. If the company
or entity seeking the opinion will not permit communication with the existing accountant, a
professional accountant in public practice shall determine whether, taking all the circumstances
into account, it is appropriate to provide the opinion sought.

b) The various kinds of threats as explained in Section 200 of ICAN code of ethics are as follows:

Self Interest Threat: The threat which occur when an auditing firm, its partner or associate
could benefit from a financial interest in an audit client Examples include direct financial
interest or materially significant indirect financial interest in a client, loan or guarantee from
concerned client, close business relationship with an audit client, potential employment with
client.

Self-Review Threat: The threat that a professional accountant will inappropriately evaluate the
result of a previous judgment made or service performed by the professional accountant or
another individual within the professional accountant‘s firm or employing organization on
which the accountant will rely when forming a judgement as part of providing a current service.
For eg: valuation service along with audit service, accounting service.

Advocacy Threat: The threat that a professional accountant will promote client‘s opinion to the
point where people may believe that objectivity is getting compromised. For eg: Auditor acting
as an advocate on behalf of audit client in litigation or dispute with the third parties.

Familiarity Threat: The threat that a profession accountant due to a long or close relationship
with a client, will be too sympathetic to their interests or too accepting of their work. For eg:
participation in client‘s affair, family and personal relationship, audit partners leaving to join
the clients etc.

Intimidation Threat: The threat that a professional accountant will be deterred from acting
objectively because of actual or perceived pressure, including attempts to exercise undue
influence over professional accountant. For eg: A firm being treated with dismissal from a
client engagement.

c) The council vide its meeting no 194 dated Falgun 4, 2071 has amended provision relating to
ceiling over the number of audit. The amended provision is as below and is effective for
appointment from Shrawan 1, 2072.

A member holding Certificate of Practice of can perform the audit of not more than 100 entities
in a financial year, out of which audit of public companies shall not exceed 10. The above limit
is applicable for each member of a partnership firm. However, audit of certain entity having
turnover of less than 20 lacs is not included while calculating the above limit.

Thus, the firm can take maximum of 200 audits (100*2), but the number of public company audit
can‘t exceed 20. As, the firm is already holding audit of 20 public limited companies Pradhan,
Ghimire & Associates can‘t take the audit of the Premier Investment Limited.

5. Answer the following: (25=10)


a) What are qualities of statutory auditor.
b) Explain the use of written representation as audit evidence, including its limitations.

Answer:
a) An auditor should adhere to the fundamental principles mentioned in code of ethics. The
fundamental principles are integrity, objectivity, professional competence and due care,
confidentiality and professional behavior. Auditor should be honest, sincere and
straightforward while performing the professional duties. Further, the qualities that an auditor
must possessed are independence, knowledge, technical skills, tact, caution, firmness, good
temper, discretion, industry judgment, patience, clear headedness and good communication
skills. He/She must be able to analyse and interpret the problems so that the situations can be
handled accordingly.

He/she must have a thorough knowledge client‘s business and other disciplines like general
principles of law, economics, taxation, information technology, management, financial
management.

b) The auditor should seek written representation as evidence that management acknowledges its
responsibility for the fair presentation of financial statement. The auditor should obtain written
representations from management on matters material to the financial statement when other
sufficient appropriate audit evidence cannot reasonably expect to exist. Further, the possibility
of misunderstanding between the auditor and management is reduced when oral representations
are confirmed by management in writing. The written representation shall be in the form of
letter addressed to the auditor. If the auditor has concerns about the competence, integrity or
diligence of management, the auditor shall determine their effect on the reliability of written
representation.
However, although written representations are a form of audit evidence, they do have their
limitations. Although written representations provide necessary evidence, they do not provide
sufficient appropriate audit evidence on their own. Furthermore, the fact that management has
provided reliable written representation does not affect the nature or extent of other audit
evidence that the auditor obtains.

6. Write short notes on the following: (42.5=10)


a) Impairment of Assets
b) Audit Note-book
c) Subsequent Events
d) Vouching of Advances to the Suppliers
Answer:
a) Impairment of Assets: Besides charging annual depreciation on assets by the reason of normal
wear and tear, affluxion of time and obsolescence to re-instate the correct value of the assets
considering the future cash flows that the assets can generate, impairment loss needs to be
provided. The difference between the carrying amount of an asset and recoverable amount is
termed as impairment loss. The treatment of impairment loss is similar to depreciation except
the fact that it can be re-instated in future, if the recoverable amount of the asset exceeds
the carrying amount. The auditor must ensure that the provisions of NAS 36 ―Impairment of
Assets‖ are followed.

a) Audit Note-book: An audit note book is usually a bound book in which a large variety of
matters observed during the course of audit are recorded. Audit note books form part of audit
working papers and for each year a fresh audit note book is maintained. In case an auditor
classifies his working paper into permanent and current, then audit note book shall form part of
the current file. It is in any case a part of the permanent record of the auditor available for
reference later on, if required.
The audit note book also provides a valuable help to the auditor in picking up the links of work
when the concerned assistant is away or the work is stopped temporarily. It is also used for
recording the various queries raised in the course of the work and their state of disposal. In
respect of disposed queries, explanation obtained and evidence seen would be recorded in the
said book, while queries remaining un-disposed of would be noted for follow up.

b) Subsequent Events: NSA 560 on ―Subsequent Events‖, defines the term ―subsequent events‖ as
events occurring between the date of the financial statements and the date of the auditor‘s
report, and facts that become known to the auditor after the date of the auditor‘s report.,
―subsequent events‖ also refer to significant events which occurred upto the date of report of
the auditor of that component. Thus, subsequent events are those events which occur after the
date of the balance sheet till the audit report is signed by the auditor.

c) Vouching of Advances to suppliers can be done as follows:

 Obtain schedule of debit balances in trade payables‘ account and pay particular attention to
the age of the balances. Also scrutinize the bought ledger.
 Enquiry should be made for long unadjusted outstanding and check as to whether any of
them would require provisioning.
 Examine that the advances have not been shown as deposits in balance sheet.
 Confirmation of balances should be obtained and reconciliation be done in case of any
discrepancies.

7. Distinguish between: (25=10)


a) Internal Check and Internal Audit
b) Vouching and Verification
Answer:
Distinction between Internal Check & Internal Audit

Internal Check Internal Audit


SN SN
1 Internal check is not a specific check, 1 Internal audit is specifically done to
but the duties of different persons are check that the accounts are properly
so arranged that a person‘s work is maintained and the systems are in
automatically checked by another control.
person while carrying out the normal
duty.
2 Internal check does the preventive job 2 Internal audit does the detective job of
i.e. internal check is derived so that identifying frauds and errors and
frauds and errors are prevented. rectifying them.
3 It is more of process in a day to day 3 It is specific defined job.
functioning of the business.
4 All the persons in the organization are 4 Specific persons are appointed to the
involved to maintain the internal internal audit.
check system.
5 It is required in all organization in 5 Carrying out internal audit is not
formal or informal. compulsory. It is done based on
management decision.
6 It does not include internal audit. 6 It include internal check.

d) Distinction between Vouching & Verification:

Points of diff. Vouching Verification


Meaning The act of examining the vouchers is Verification van be explained as
known as vouching. A voucher is any establishing the truth or securing
documentary evidence in support of a some kind of confirmation with
transaction entered in the books of account.
respect to the assets and
liabilities appearing in the
Balance Sheet of a concern.
Nature Vouching involves establishing the Verification goes beyond
& arithmetical accuracy and the authenticity vouching. It seeks to establish
Purpose ofthe transactions of a concern. Vouching that assets as stated in the
proves that an asset ought to exist. Balance Sheet of a concern exist
in fact and that the liabilities are
properly disclosed. Verification
proves that an asset does exist.
Time It is done during the whole year. It is done during end of the year.
Utility Certifies correctness of records. Certifies correctness of assets
and liabilities.
Personnel It is done by the junior staff of the auditor It is done by the auditor himself
under the supervision of a senior person. assisted by senior.
Corporate & Other Law
Maximum Marks - 100

Total No. of Questions - 7 Total No. of Printed Pages -11


Time Allowed - 3 Hours
Marks
Attempt all questions.
1. Answer the following questions: (5×5=25)
a) Mount Everest Ltd. passed a special resolution at the meeting of the
shareholders to issue shares at a discount to a commercial bank. The objective
of the issue is to convert the loan borrowed by the company from the
commercial bank to equity share. Discuss the validity of resolution passed in
the context of the provisions regarding prohibition to issue shares at a discount
as specified in the Companies Act, 2063?
b) The meeting of the Board of Directors of Royale Tours Ltd. decided to issue
preference share as it was authorised by its memorandum and articles of
association. Advise them regarding the matters to be disclosed while issuing
the preferential shares.
c) Mr. Harry is in the process of incorporating public company in association
with his friends. But he is not aware about the validity of different expenses
done by him during the process of incorporating companies and wants your
expert advice on this. Suggest him the relevant provision of the Companies
Act, 2063 about the pre-incorporation contract.
d) 'Doctrine of indoor management can't be claimed by an outsider dealing with
the company in every circumstances'. Justify your answer.
e) CEO of the Reliance Hydropower Limited is in the process to sue against the
supplier of the Electro mechanical equipment to the company for breach of the
contract. But he is not sure about the legal provision of the Companies Act
whether the company can sue or not. Advise the CEO of the Hydropower
company regarding the relevant provision of the Companies Act whether the
company can sue or not?
Answer:
a) Section 64 of Company Act, 2063 makes provision regarding the prohibition on issue or
sale of share at a discount. Provision of Section 64 of Company Act, 2063 is as follows:
Section 64 - Prohibition on issue or sale of share at a discount
(1) A Company shall not issue or sale its share at a discount.
(2) Notwithstanding anything contained in sub-section (1), a company may, for the following
purpose, issue or sell shares at discount by adopting a special resolution at the General
Meeting to that effect, not being less than the percentage specified in that resolution. (3
for a, b, c, d)
(a) for issuing or selling shares pursuant to a capital restructuring scheme of the
company.
(b) for issuing or selling shares pursuant to a scheme of converting loans borrowed by
the company into shares with the consent of creditors.
(c) for issuing or selling shares pursuant to an employee share scheme.
(d) for issuing shares on such other conditions/purposes as approved by the office.

As the company has passed a special resolution to convert the loan borrowed by the company
from the commercial bank to equity share, resolution passed is valid as per the provision
above mentioned.

b) A company may issue preference shares as provided for in the Companies Act, 2063,
memorandum or articles of association. While issuing preference share, the issuing
company must disclose the various matters as prescribed by the Act. As per Section
65(3), the following maters shall be disclosed:
 Whether preference is given to receive dividends against ordinary shares;
 Percentage of dividends receivable by preference shareholders;
 Whether dividends get cumulated every year (cumulative) or profits are distributed
only in a year wherein profit is made (non-cumulative);
 Whether preference is given while paying amount of share in the event of liquidation
of company;
 Whether voting right is attached there to; and if voting right is attached, whether such
right is available only in the case of preference share or also in other matters;
 Whether voting right is available also in other matters pursuant to Clause (e), the
proportion to which such right is exercisable;
 Whether preference shares can be converted into ordinary shares;
 Whether the amount of preference shares can be redeemed (redeemable) or cannot be
redeemed (irredeemable) after a certain period;
 Whether in redeeming preference shares, premium is payable on redemption.

c) As per section 17 of Companies Act, 2063


(1) A contract made prior to the incorporation of a company shall be a proposed contract
only, and such contract shall not be binding on the company.
(2) If, prior to the incorporation of a company, any person carries on any transaction or
borrows money on behalf of the company, such person shall be personally liable for
any contract related with the transaction so carried on, subject to Sub-section (3).
(3) If, within the time mentioned in any transactions or within the reasonable time after
the incorporation of a company, the company, through its act, action or conduct,
accepts any act, action or conduct, accepts any act, action to borrowing done or made
prior to the date of authorization to commence its transactions or endorses such act or
action, that transaction shall be binding on the company and the other contracting
party; and the person carrying out such act to action shall be released from the
personal liability to be borne pursuant to Sub-section(2).
(4) Notwithstanding anything contained elsewhere in this Section, the consensus
agreement of a private company shall govern any contracts made prior to the
incorporation of such company.
d) The rule of doctrine of indoor management is, however, subject to certain exceptions. In
other words, relief on the ground of ‗indoor management‘ can't be claimed by an outsider
dealing with the company in the following circumstances:
1. Knowledge of Irregularity: -
The first and the most obvious restriction is that the rule has no application where the
party affected by an irregularity had actual notice of it. Knowledge of an irregularity
may arise from the fact that the person contracting was himself a party to the inside
procedure. Similarly, in Howard v. Patent Ivory Manufacturing Co.[10], where the
directors could not defend the issue of debentures to themselves because they should
have known that the extent to which they were lending money to the company
required the assent of the general meeting which they had not obtained.
2. Suspicion of Irregularity: -
The protection of the ―Turquand Rule‖ is also not available where the circumstances
surrounding the contract are suspicious and therefore invite inquiry. Suspicion should
arise, for example, from the fact that an officer is purporting to act in matter, which is
apparently outside the scope of his authority.
3. Forgery: -
Forgery may in circumstances exclude the ‗Turquand Rule‘. The only clear
illustration is found in the Ruben v Great Fingall Consolidates [15]; here in this
case the plaintiff was the transferee of a share certificate issued under the seal of the
defendant‘s company. The company‘s secretary, who had affixed the seal of the
company and forged the signature of the two directors, issued the certificate.
4. Representation through Articles: -
The exception deals with the most controversial and highly confusing aspect of the
―Turquand Rule‖. Articles of association generally contain what is called ‗power of
delegation‘.
The effect of a ―delegation clause‖ is ―that a person who contracts with an individual
director of a company, knowing that the board has power to delegate its authority to
such an individual, may assume that the power of delegation has been exercised.‖
5. Acts outside apparent authority: -
Lastly, if he act as an officer of a company is one which would ordinarily be beyond
the power of such an officer, the plaintiff cannot claim the protection of the
―Turquand rule‖ simply because under the articles power to do the act could have
been delegated to him. In such a case the plaintiff cannot sue the company unless the
power has, in fact, been delegated to the officer with whom he dealt. A clear
illustration is Anand Behari Lal v Dinshaw[18] here the plaintiff accepted a transfer
of a company‘s property from its accountant. Since such a transaction is apparently
beyond the scope of an accountant‘s authority‘ it was void. Not even a ‗delegation
clause‘ in the articles could have validated it, unless he was, in fact, authorized.

e) As per the section 7 of the Companies Act, 2063.


(1) Any company incorporated under this Act shall be an autonomous and corporate
body with perpetual succession.
(2) Subject to this Act, company like an individual, acquire, hold, sell, dispose of or
otherwise deal with, any movable or immovable property.
(3) A company may sue and be also sued by its own name.
(4) A company may, like an individual, enter into a contract and exercise the rights and
perform the obligations as referred to in the contract.
Hence as per the above mentioned provision of the companies Act, Reliance
Hydropower can sue against the supplier on its own.
2. Answer the following questions: (3×5=15)
a) ABC Bank Limited (proposed) has filed an application to Nepal Rastra Bank
(NRB) for the prior approval of incorporation of the bank. NRB has refused
to grant approval citing that name of the proposed bank or financial
institution is identical with the name of another bank in existence. Give your
view on the contention of the NRB referring relevant provision of the Banks
and Financial Institutions Act (BAFIA), 2073.
b) What are the objectives of the Nepal Rastra Bank as per the Nepal Rastra
Bank Act, 2058?
c) Miss Dakshina has been elected as a Board of Director of Garima Bank Ltd.
You, an employee of the bank has to prepare a document including the
matters to be submitted by a board of director after her appointment. List out
the matters you have to collect from her for the disclosure to be made by the
director as per the Banks and Financial Institutions Act (BAFIA), 2073.
Answer:
a) As per section 7 (1)(a) of the Banks and Financial Institution Act, 2073; The Rastra Bank
may refuse to grant prior approval for the incorporation of a bank or financial institution,
if the name or the operation of the proposed bank or financial institution is not in the
interest of the public at large, religion or community, etc.
In the given case the name of the proposed bank is identical with the name of the another
bank in existence which may not be in the interest of public at large. Hence the refusal of
the NRB to grant approval in the above mentioned case is in line with the section 7(1)(ka)
of the Banking and Financial Institution Act, 2073.

b) As per Section 4(1) of Nepal Rastra Bank Act, 2058, the objectives of the Nepal Rastra
Bank shall be as follows:-
(a) To formulate necessary monetary and foreign exchange policies in order to maintain
the stability in the price and balance of payment for financial stability and sustainable
development of economy, and manage it;
(b) To promote entire banking and financial system of the Nepal and to enhance its public
credibility by maintaining stability in banking and financial sector;
(c) To develop a secure, healthy and efficient system of payment;

Also as per the section 4(2), the Bank shall, without any prejudice to the objectives
referred to in Sub-section (1), extend co-operation in the implementation of the economic
policies of Government of Nepal.

c) According to Section 24 of the BAFIA, 2073 has provided the matters to be disclosed after
assuming the office by every director. As per the section, every director shall disclose in
writing to the bank or financial institution the following matters:
1. Full name, address, academic qualification and experience of director.
2. If he or she is a director, office bearer or employee of any other institution, the
details of such post and responsibility thereof.
3. If he or she or any of his or her family members has entered into or going to enter
into any kind of contract with the concerned bank or financial institution, details
thereof.
4. If he or she has any kind of interest in the appointment of the chief executive,
managing director, secretary, auditor and general manager, details thereof.
5. Particulars of such shares or debentures in the concerned bank or financial institution
or in its holding or subsidiary company as subscribed by him or her or by his or her
family.
6. If he or she is a director of any company, details thereof.
7. If any member of his or her family is working as an officer of the bank or financial
institution, details thereof.
8. Power of Attorney given to the Rastra Bank to or caused to enquiry regarding the
director's financial and professional background or to or caused to exchange of such
notice or information.
9. Self-declaration of his eligibility to be a director as per the Act.
10. Such other details prescribed by the Rastra Bank as required to be disclosed by the
director to the Board.

3. Answer the following questions: (2×5=10)


a) Mr. Pradip Baral, one of the directors of XYZ Bank Limited passes some
secret information regarding the securities of the Bank to his brother. On the
basis of that secret information his brother bought huge number of shares of
XYZ Bank. Few months later, share price went up and Mr. Pradip‘s brother
earned huge amount of money from the transaction of those shares. Did Mr.
Pradip commit any crime under the Securities Act, 2063? If yes, what crime
did he commit?
b) State the grounds under which the Insurance Board may cancel the registration
of an insurance company (insurer) under the Insurance Act, 2049.
Answer:
a) Yes, Mr. Pradip Baral committed a crime against the Securities Act, 2063. His activities
are known as insider trading and it is punishable crime under Securities Act, 2063.
Section 91 of the Securities Act, 2063 states that in case any person conducts or causes
others to deal in securities on the basis of any insider information or notice that are
unpublished or communicates any information or notice known to such a person except in
the course of the discharge of his or her duties in manner likely to affect the price of
securities such a person shall be deemed to have been committed an insider trading in
securities.
b) Section 13 of the Insurance Act, 2049 provides the grounds for the cancellation of
registration of insurer by the Insurance Board.
As per section 13 of Insurance Act, the Insurance Board may cancel the registration of an
insurer by providing a written notice with effect from the date prescribed in the same
notice in the following circumstances:
a) If the insurance business is not started within six months from the date of obtaining the
certificate.
b) If it is felt that the liability of the insurer exceeds its assets within Nepal.
c) If the insurer could not fulfill the liability pursuant to the decision within three months
from the date of final decision of the court in the case filed under the insurance policy
issued within Nepal.
d) If the head office of the insurance business of any foreign insurer is situated outside
Nepal and in case it felt that Nepalese insurer has not obtained equal facilities there
which are enjoyed by the foreign insurer pursuant to the prevailing law of such
company.
e) If the insurer does not open its office inside Nepal.
f) If the insurer does not perform the functions to be performed or has performed any
functions which is not to be performed pursuant to this Act or the rule made under this
Act.

Before cancelling the registration of an insurer pursuant to sub-section 1) above, the


Insurance Board shall provide a reasonable time-limit to submit clarification to the
concerned insurer, stating the reason for cancelling its registration.
If the concerned insurer does not submit its clarification within the time period
mentioned in sub-section 2) above or clarification submitted by it is found not to be
satisfactory, the Insurance Board shall cancel the registration of such insurer pursuant
to sub-section 1) above, and shall publish a notice in two major newspapers to be
published in Nepal for the public information in general.
4. Answer the following questions: (2×5=10)
a) Discuss the functions, duties and power of Industrial Promotion Board
specified under the Industrial Enterprises Act, 2073.
b) Mentioned the eligibility criteria for entitlement of the bonus by the employee
as per the Bonus Act, 2030.

Answer:
a) The Functions, Duties and Power of the Board are listed in the section 19(1) of Industrial
Enterprise Act, 2073. Major functions, duties and power of the Board are as follows:-
(a) To make policy decision regarding industrial promotion, safeguarding and promoting
of investment and industrialization.
(b) To make necessary cooperation in implementing policies, laws and regulations
pertaining to the industry.
(c) To make regular evaluation of the policy related, legal, institutional and procedural
structure for industrialization of the whole country and recommend to the
Government of Nepal for necessary amendment in those areas.
(d) To make necessary policy decision on foreign investment and technology transfer
based upon the related existing laws.
(e) To make policy decision for the prevention of the industrial pollution.
(f) To carry out evaluation and review of development of industrial sector of the country
and make recommendation to Government of Nepal to take necessary steps.
(g) To address the issues raise while providing service, facilities and concession to the
industry as per this act and other related acts.
(h) To resolve the complain made by the industrialist by giving directives to the
concerned body.
(i) To make recommendation to Government of Nepal for the change of level,
classification and nature of any industries.
(j) To recommend the Government of Nepal to carry out necessary study, research and
survey for the promotion and development Industrial and foreign investment.

b) As per section 6 of the Bonus Act, 2030;


(1) An employee who has worked at least half period in a fiscal year, shall be entitled to
obtain bonus underthis Act. Provided that, no employee shall be entitled to obtain Bonus
who hasworked casually or in a shift basis.
(2) For the purpose of Sub-section (1), the following periods shall alsobe computed as a
period where an employee has worked.

(a) A period kept on reserve under any contract or under Section 11 of the Labour Act,
2048.
(b) A period under which an employee is on any leave with salary.
(c) A period of disablement caused by accident arising in course of business of the
enterprise.

5. List out the objectives of the ICAN and state the formation of council
including the election of president and vice-president pursuant to the Nepal
Chartered Accountants Act, 2053. (4+3+3=10)
Answer:
Objectives of ICAN:
ICAN has an important role to make accounting profession as reliable, authentic and standard
for its accuracy and enhance economic and social responsibility as a cause for economic
development of the nation. As per Section 5 of the NCA Act, 2053, the objectives of the
Institute shall be as follows:-
1. To play the role of a regulatory body to encourage the members to carry on accounting
profession being within the extent of the code of conduct in order to consolidate and
develop accounting profession as a cause for economic development of the nation.
2. To enhance social recognition and faith in accounting profession by raising awareness of
the general public towards the importance of accounting profession and the economic and
social responsibility of professional accountants.
3. To develop, protect and promote the accounting profession by enabling professional
accountants understand their responsibility towards the importance of accounting
profession and accountancy.
4. To develop mechanism of registration, evaluation and examination of accounting
professionals in consonance with international norms and practices so as to make the
accounting profession respectable and reliable.
Formation of the Council:
Pursuant to Section 7 (3), the Council shall consist of the following Council members:-
 Ten persons elected by and amongst Chartered Accountant members -
Member
 Four persons elected by and amongst Registered Auditors -
Member
 Three persons nominated by Government of Nepal, upon the recommendation of the
Auditor General, from amongst the persons well experienced in the field of accounting
profession. – Member
Election of President and a Vice-President:
Sub-section (4) provides the election of the President and a Vice-President. It reads that
the Council members shall elect a President and a Vice-President from the Fellow
Chartered Accountants (FCA) Council members referred to in clause (a) of sub-
section (3). The term of office of the chairperson and the vice- chairperson shall be
one year; and upon the expiry of their term of office, they may be elected for one
more term.

6. Answer the following: (5×4=20)


a) What are the essential elements of valid contract?
b) As per Securities Act, 2063 what matters need to be mentioned in prospectus
and in what conditions issuance of prospectus is not necessary?
c) What are the Functions, duties and powers of Accounting Standards Board
under the Nepal Chartered Accountants Act, 2053?
d) Mentions the major functions of World Trade Organization (WTO).
e) What special programs may be operated by the Government of Nepal relating
to the social welfare as per Social Welfare Act, 2049?

Answer:
a) The essential elements of valid contract are as follows
i) Proper offer and acceptance
There must be an agreement between two parties to create a contract. The agreement
involves a valid offer by one party and valid acceptance of the offer by the other party.
ii) Legal relationship
The parties of a contract must have to intend that it creates legal relation and has legal
consequences.
iii) Free consent
Free consent of the contracting parties is a must to create a contract. Consent may not
be free if it is obtained on the ground of coercion, undue influence, fraud,
misrepresentation and mistake of law.
iv) Meeting of minds
Meeting of minds (consensus ad idem) is necessary for a valid contract. The parties are
said to be of meeting of mind when they give their consent on the subject matter of the
contract in same sense and at the same time.
v) Competent parties
The parties to the agreement must be competent to contract. If incompetent parties are
in contract, it is not valid. eg. Minor
vi) Lawful objectives
The objective of a contract must be lawful. Those objectives are not lawful if they are
illegal, immoral and opposed to public policy.
vii) Lawful consideration
The agreement is enforceable only when contracting parties are giving and getting
something in return. The consideration must be lawful, which may be in past, present
or future.
viii) Agreement not declared void
The agreement must not be expressly declared void by any existing law.
ix) Certainty of meaning
Agreement must be certain and not vague or indefinite or words of latent ambiguity.
x) Possibility of performance
The objective or terms of contract must be possible to perform. Performance is the
very nature of contract, so impossible contract are not valid.
xi) Legal formalities
A contract may be made expressly but some contracts are recognized after fulfilling
certain formalities.

b) As per Section 32 of Securities Act, 2063 every prospectus shall contain such general
matters as required to be set down in the prospectus, capital and other information of
the issuer, main functions to be done by the issuer, information pertaining to legal
action, economic condition, general administration, management of the issuer,
information relating to the expert preparing the prospectus and the economic
statements contained in the prospectus and such other matters as may be prescribed.
In case of securities issued by the Nepal Rastra Bank, securities issued against the full
guarantee of the Government of Nepal, securities proposed to be sold to up to fifty
persons at a time, securities issued to own workers or employees, and securities
permitted by the SEBON as to issue and sell without issuing a prospectus.
c) Pursuant to Section 15a of the Nepal Chartered Accountant Act 2053, Government of
Nepal shall form a Accounting Standard Board in order to govern and regulate
financial reporting and accounting profession under the chairmanship of a person
nominated by Government of Nepal from amongst the fellow chartered accountants.
The Functions, duties and powers of Accounting Standards Board to be formed
pursuant to Section 15a are given under Section 15b of the Nepal Chartered
Accountants Act, 2053 as follows:
i) To develop Accounting Standards, on the basis of relevant International
Accounting Standards, in order to govern and regulate financial reporting and
accounting profession;
ii) To evolve appropriate process of development of accounting standards and publish
materials relating to accounting standards;
iii) To redraft, improvise and revise standards;
iv) To interpret the standards;
v) To undertake other related tasks related to accounting standards.

d) The major functions of WTO are discussed as below:


1) Administering WTO agreements:
The WTO agreements cover goods, services and intellectual property. They
include individual countries' commitments to lower customs tariffs and other
trade barriers and to open and keep open services markets. It has different
mechanism like General Council which works on behalf of ministerial
conference. It meets to Dispute Settlement Body and Trade Policy Review Body
to oversee procedures for settling disputes and to analyze members' trade
policies. There are Goods Council, Services Council and TRIPS Council with
various committees to works on related sectors. The ministerial conference can
take decisions on all matters under any of the multilateral trade agreements.

2) Forum for trade negotiation:


It provides forum for trade negotiation. For this purpose, its different mechanism
activate to works for their responsible sectors. Issues on trade related aspects
could be submitted through committees and councils for negotiations.

3) Handling trade disputes:


It has dispute settlement mechanism like Dispute Settlement panels and Dispute
Settlement Body (General Council in another guise). It is under the General
Council and finally ministerial conference. The Dispute Settlement Body has the
sole authority to establish ―panels‖ of experts to consider the case, and to accept
or reject the panels‘ findings or the results of an appeal. It monitors the
implementation of the rulings and recommendations, and has the power to
authorize retaliation when a country does not comply with a ruling.
4) Monitoring national trade policies:
It monitors national trade policies through General Council Meetings as Trade
Policy Review Body. Finally, the general council submits the report before
conference.
5) Technical assistance and training for developing countries:
It provides technical assistance and training as it think fit. It has technical
assistance missions that works in this field.
6) Cooperation with other international organizations:
It cooperates with and assist to IMF and IBRD for establishing coherence in
universal economic policy determination. The WTO maintains extensive
institutional relations with many of its sister organizations, participates as
observer in their work and has established several partnerships to help improve
the trading opportunities and capacities of developing and least-developed
countries. Examples of such partnerships are the Enhanced Integrated Framework
(EIF), the Standards and Trade Development Facility (STDF) and the Aid for
Trade Initiative. WTO cooperation with other international organizations
continues to evolve and is more than ever a function of the need for increased
global coordination and better governance.

e) Pursuant to Section 3 of the Social Welfare Act, the Government of Nepal, by means
of different activities relating to the social welfare work, to support the overall
development of the country may operate the social welfare Program through the
concerned Ministry and Social organizations and institutions.
Similarly, as per Section 4, the Government of Nepal may operate special Programs,
relating to the social welfare activity and social service, in the following matters:
(a) To serve interest and render welfare to the children, old age, helpless or disabled
people.
(b) To foster participation in development and to promote and protect the welfare,
rights and interest of the women.
(c) To rehabilitate and help to lead a life of dignity to the victims of social mischief's
and also to juvenile delinquency, drug addicts and similar people involved in other
kind of addictions.
(d) To help to lead a life with dignity to the jobless, poor and illiterate people.
(e) To manage religious places and the activities of the trust Guthi institutions.
(f) To take effective management and actions for the welfare of the backward
communities and classes.
7. Write short notes on the following: (2×5=10)
a) Doctrine of unjust enrichment
b) Formation of Insurance Board
Answer:
a) A general equitable principle that no person should be allowed to profit at another's
expense without making restitution for the reasonable value of any property, services,
or other benefits that have been unfairly received and retained.
Although the unjust enrichment doctrine is sometimes referred to as a quasi-
contractual remedy, unjust enrichment is not based on an express contract. Instead,
litigants normally resort to the remedy of unjust enrichment when they have not
written or verbal contract to support their claim for relief. In such instances litigant
ask a court to find a contractual relationship that is implied in law, a fictitious
relationship created by courts to do justice in a particular case.
Its objective is to prevent a man from retaining the money of or some benefit derived
from another, which it is against conscience that he should keep.
The doctrine is applied in the satisfaction of the following conditions:
1. The plaintiff must have provided the defendant with something of value while
expecting compensation in return
2. The defendant must have acknowledged, accepted, and benefited from whatever
the plaintiff provided.
3. The plaintiff must show that it would be inequitable or unconscionable for the
defendant to enjoy the benefit of the plaintiff's actions without paying for it.
A court will closely examine the facts of each case before awarding this remedy and
will deny claims for unjust enrichment that frustrate public policy or violate the law.

b)
Section 3 of Insurance Act, 2049 has provided the provision of formation of the
Insurance Board. It is formed to systematize, regularize, develop and regulate the
Insurance Business.
The Board pursuant to Sub-section (1) shall consists of the following Members:
(a) A person nominated or designated by the Nepal Government – Chairperson
(b) Representative, Ministry of Law, Justice and Parliamentary Affairs – Member
(c) Representative, Ministry of Finance – Member
(d) A person nominated by the Nepal Government from among the persons having
the special knowledge in the Insurance Business – Member
(e) A person nominated by the Nepal Government from among the Insured –
Member
An employee designated by the Board shall perform the duty as a Secretary of the
Board.
The Nepal Government may make alteration of the Members of the Board by
publishing a notification in the Nepal Gazette, if it deems necessary.
If it is deemed necessary, the Board may invite any national or foreign experts in the
meeting of the Board as an observer.
The tenure of the nominated Members of the Board shall be four years. They may be
re-nominated up to twice after the expiry of their tenure.
The Head office of the Board shall be located in Kathmandu.
Financial Management
Maximum Marks - 100
Total No. of Questions – 7 Total No. of Printed Pages – 14
Time Allowed – 3 Hours
Marks
Attempt all questions.
Working notes should form part of the answer. Make assumptions wherever necessary.
1.
a) Ganesh Enterprises needs someone to supply it with 150,000 cartons of
machine screws per year to support its manufacturing needs over the next five
years, and you‘ve decided to bid on the contract. It will cost you Rs. 780,000
to install the equipment necessary to start production; you‘ll depreciate this
cost straight-line to zero over the project‘s life. You estimate that in five years
this equipment can be salvaged for Rs. 50,000. Your fixed production costs
will be Rs. 240,000 per year, and your variable production costs will be Rs.
8.50 per carton. You also need an initial investment in net working capital of
Rs. 75,000. Your tax rate is 35 percent and you require a 16 percent return on
your investment.
Required: 10
Calculate unit bid price you should submit.

b) Consider the situation of a company, BIKE WASH, which must decide


whether to replace an existing machine. BIKE WASH currently pays no
taxes. The replacement machine costs Rs. 9,000 now and requires
maintenance of Rs. 1,000 at the end of every year for eight years. At the end
of eight years, the machine would be sold for Rs. 2,000 after taxes. The
existing machine requires increasing amounts of maintenance each year, and
its salvage value falls each year as below:
Year Maintenance (Rs.) After tax Salvage (Rs.)
Present 0 3,000
1 1,000 2,500
2 2,000 1,500
3 3,000 1,000
4 4,000 0
The existing machine can be sold for Rs. 3,000 now after taxes. If it is sold
one year from now, the resale price will be Rs. 2,500 after taxes, and Rs.
1,000 must be spent on maintenance during the year to keep it running.
Assume that this maintenance fee is paid at the end of the year. The machine
will last for four more years before it falls apart with zero salvage value at the
end of year 4. BIKE WASH faces an opportunity cost of capital of 15 percent.
Required: 10
Determine when BIKE WASH should replace the machine.
Answer:
a)
i) Initial Outlay
Cost of the Machine 780,000.00
W/C Infusion 75,000.00
855,000.00

ii) Annual Depreciation and Tax Shield


Depreciable Value 780,000.00
Life 5 Yrs
Annual Depreciation 156,000.00
Tax Shield on Annual Depreciation [Depreciation ×
Tax Rate] 54,600.00

iii) Terminal Value


Net proceed from the Sale of Machine 32,500.00
[50,000-(50,000-0)×35%]
Release of Working Capital 75,000.00
Total Terminal Value 107,500.00

iv) Total Cash Cost Per Annum - Post Tax


Annual Variable Cost (150000×Rs.8.50) 1,275,000.00
Annual Fixed Cost 240,000.00
Total annual Cash Cost 1,515,000.00
Tax @ 35% 530,250.00
Annual Post Tax Cash Cost 984,750.00

v) Suppose SP Per Cartoon X


Annual Revenue from Sale of 150,000 Cartoon 150,000 X
Post Tax Revenue from Sale 97,500 X

vi) Net Present Value Table


Particulars Time Cash Flow PVF @ 16% Total PV
Initial Outlay 0 (855,000.00) 1.0000 (855,000.00)
Tax Shield on Annual Depreciation 1-5 54,600.00 3.2743 178,777.00
Terminal Cash Flow 5 107,500.00 0.4761 51,181.00
Post Tax Cash Cost 1-5 (984,750.00) 3.2743 (3,224,367.00)
Post Tax Revenue 1-5 97,500 X 3.2743 319,244.00 X
Net Present Value 0

Since the discount rate 16% incorporates the profit target as well, the required NPV of the Project is
Zero.

Now,
319,244.00X+51181.00+178,777.00=855,000+3,224,367.00
319,244.00X= 3,849,409.00
X= 12.06

Therefore the Bid Price per Cartoon is Rs. 12.06

b)
Our decision will be based on the comparison of the annual cost of the replacement
machine with the annual cost of the old machine.

i) Equivalent Annual Cost of New Machine


The present value of the cost of the new replacement machine is as follows;
Present Value = 9,000 + 1,000 × [PVIFA, 8 Years, 15%] – 2,000× [PVIF, 8th Year, 15%]
= 9,000 + 1,000×4.4873 – 2,000×0.3269
= Rs. 12,833.5
[1]
The Equivalent Annual Cost of new replacement machine equals:
= Present Value / 8 Year annuity factor at 15%
= 12,833.5/4.4873
= Rs. 2,860
This calculation implies that buying a replacement machine is financially equivalent to renting
this machine for Rs. 2,860 per year.

ii) Cost of Old Machine


If BIKE WASH keeps the old machine for one year, the firm must pay maintenance costs of Rs.
1,000 a year from now. BIKE WASH will receive Rs. 2,500 at the end of year if the old machine
is kept for one year but would receive Rs. 3,000 today if the old machine were sold immediately.
This reduction in sales proceeds is clearly a cost as well. Further we normally express cash flows
in terms of present value; the analysis to come is easier if we express the cash flow in terms of its
future value one year from now.

The Value for end of year 1, 2 3 and 4 will be as below;


Particulars Year-1 Year-2 Year-3 Year-4
Beginning Salvage Value [A] 3,000 2,500 1,500 1,000
Maintenance cost expressed in 1,000/1.15 2,000/1.15 3,000/1.15 4,000/1.15
term of beginning value [B] =870 =1,739 =2,609 =3,478
End Salvage value expressed in (2,500)/1.15 (1,500)/1.15 (1,000)/1.15 -
term of beginning value [C] =(2,174) =(1,304) =(870)
Net Present Value D= [A+B-C] 1,696 2,935 3,239 4,478
Future Value [D×1.15] 1,950 3,375 3,725 5,150
[4
iii) Making the Comparison
If BIKE WASH replaces the machine immediately, we can view the annual expense as Rs. 2,860
beginning at the end of the year. This annual expense occurs forever if it replaces the new
machine every eight years.
If BIKE WASH replaces the old machine in one year, its expense from using the old machine for
that year can be viewed as Rs. 1,950 payable at the end of the year.
Therefore BIKE WASH should not replace the machine now. However, if we look into the cost
of operating old machine from 2nd year and onward, the cost is always higher than the equivalent
annual cost of new machine.
Therefore, the old machine should be replaced after operating 1st Year.

2.
a) X and Y are two fast growing companies in the engineering industry. They
are close competitors, and their asset composition, capital structure, and
profitability records have been very similar for several years. The primary
difference between the companies, from a financial management
prospective, is their dividend policy. Company X tries to maintain a non-
decreasing dividend per share, while company Y maintains a constant
dividend payment ratio. Their earnings per share (EPS), dividend per share
(DPS), and average share price are as follows:

Company X (Rs.) Company Y (Rs.)


Year EPS DPS Avg. Price EPS DPS Avg. Price
1 9.30 2.00 87.50 9.50 1.90 70.00
2 7.40 2.00 67.50 7.00 1.40 45.00
3 10.50 2.00 90.00 10.25 2.10 57.50
4 12.75 2.25 110.00 12.25 2.45 100.00
5 20.00 2.50 167.00 20.25 4.05 167.50
6 16.00 2.50 170.00 17.00 3.40 160.00
7 19.00 2.50 182.00 20.00 4.00 160.00

Required: (5+3+2=10)
i) Determine the dividend payment ratio and price earnings ratio for both
companies for all the years.
ii) Determine the average DP ratio and PE ratio for both the companies
over the period 1 through 7 years.
iii) The management of company Y is puzzled as to why their share prices
are lower than those of company X, in spite of the fact that profitability
of company Y is slightly better (particularly of past three years). As a
financial consultant, how would you explain the situation?
b) A company has a total investment of Rs. 4,000,000 in assets and 40,000
outstanding ordinary shares of Rs. 100 per share (par value). It earns at a
rate of 15 percent on its investment, and has a consistent policy of retaining
50 percent of the earnings. The appropriate discount rate of the firm is 10
percent.
Required: (3+2=5)
i) Determine the price of its share using Gordon‘s model.
ii) What shall happen to the price of the shares if the company has a payout
of 20 percent and 60 percent respectively?
Answer:
i) Calculation of DP ratio and PE ratio of Company X
Avr.
Year EPS(Rs.) DPS(Rs.) DP ratio Price(Rs.) PE ratio
1 9.30 2.00 21.50% 87.50 9.41
2 7.40 2.00 27% 67.50 9.12
3 10.50 2.00 19.05% 90.00 8.57
4 12.75 2.25 17.64% 110.00 8.63
5 20.00 2.50 12.50% 167.50 8.38
6 16.00 2.50 15.63% 170.00 10.63
7 19.00 2.50 13.16% 182.50 9.61
94.95 15.75 875

Calculation of DP ratio and PE ratio of Company Y


Avr.
Year EPS(Rs.) DPS(Rs.) DP ratio Price(Rs.) PE ratio
1 9.50 1.90 0.20 70.00 7.37
2 7.00 1.40 0.20 45.00 6.43
3 10.25 2.10 0.20 57.50 5.61
4 12.25 2.45 0.20 100.00 8.16
5 20.25 4.05 0.20 167.50 8.27
6 17.00 3.40 0.20 160.00 9.41
7 20.00 4.00 0.20 160.00 8.00
96.50 19.30 760
ii) The average DP ratio for company X
= 15.75/ 94.95×100
=16.59 %
The average DP ratio for company Y
= 19.30/ 96.50×100
=20 %
The average PE ratio for company X
= 875/94.95
= 9.22
The average PE ratio for company Y
= 760/96.50
= 7.88
iii) Company X is following the stable dividend policy, whereas company Y is
following the stable dividend payment ratio policy. In the latter policy, sporadic
dividend payment occurs, which makes its owners very uncertain about the returns
they can expect from their investment in the firm and, therefore, generally depress
the share prices. It is probably for this reason that company X‘s share price is better
than that of Y (during the last three years).
b)
(i) Price of Share using Gordon‘s model:

The share valuation model of Gordon is as follows:


P0 = DIV1 = (1 – b)EPS1 = (1 – b)rA , where
k–g k – br k – br
A denotes investment per share, which is Rs. 100 in the present case.
When the payout is 50 per cent, the price of share will be:
P0 = (1 – 0.5) 0.15 x 100 = 0.5 x 15 = 7.5/0.025 = Rs. 300
0.10 – (0.15 x 0.5) 0.10 – 0.075

(ii) Price of Share at Payout of 20 and 60 percent:


At Payout of 20 per cent

Ke =10%
b×r = 12%
Since br ˃ Ke as per Gordon's model price could not be computed.

At Payout of 60 per cent:

P0 = (1 – 0.4) 0.15 x 100 = 0.6 x 15 = 9/0.04 = Rs. 225


0.10 – (0.15 x 0.4) 0.10 – 0.06

3.
a) A Company is preparing a cash flow forecast for the three-month period
from January to the end of March. The following sales volumes have been
forecasted:
December January February March April
Sales (units) 1,200 1,250 1,300 1,400 1,500
Additional Information:
 The selling price per unit is Rs. 800 and a selling price increase of 5%
will occur in February. Sales are all on one month‘s credit.
 Production of goods for sale takes place one month before sales.
 Each unit produced requires two units of raw materials, costing Rs. 200
per unit. No raw materials inventory is held. Raw material purchases are
on one months‘ credit.
 Variable overheads and wages equal to Rs. 100 per unit are incurred
during production, and paid in the month of production.
 The opening cash balance at 1st January is expected to be Rs. 40,000.
 A long-term loan of Rs. 300,000 will be received at the beginning of
March.
 A machine costing Rs. 400,000 will be purchased for cash in March.
Required: (6+2=8)
i) Calculate the cash balance at the end of each month in the three-month
period.
ii) Calculate the forecast current ratio at the end of the three-month period.

b) Future Kidd Corporation presently gives credit terms of 'net 30 days'. It has
Rs. 60 Million in credit sales and its average collection period is 45 days.
To stimulate sales, the company may give credit terms of 'net 60 days'. If it
does instigate these terms, sales are expected to be increased by 15%. After
the change, the average collection period is expected to be 75 days with no
difference in payment habits between old and new customers.
Variable cost is Re. 0.80 for every Re 1.00 of sales and the company's
before tax required rate of return on investment in receivables is 20%.
Required:
Should the company extend its credit period? (Assume a 360-days year) 7
Answer:
a) (i)
Monthly Sales
Particulars December January February March April

Sales (units) 1,200 1,250 1,300 1,400 1,500

Selling price (Rs./unit) 800 800 840 840

Sales (Rs.000) 960 1,000 1,092 1,176

Purchases:
December January February March April

Production units 1,250 1,300 1,400 1,500


Raw materials (units) 2,500 2,600 2,800 3,000
(2×Production unit)
Raw materials (Rs. 000) 500 520 560 600
(@Rs.200/unit)
Variable Cost:
December January February March April

Production (units) 1,250 1,300 1,400 1,500


Variable costs (Rs.000) 125 130 140 150
(@Rs. 100/unit)

Monthly cash balances:

Particulars January (Rs. 000) February (Rs. 000) March (Rs. 000)
Cash collection from 960 1,000 1,092
Receivables
Loan 300
Total receipts 960 1,000 1,392
Payments:
Raw materials 500 520 560
Variable costs 130 140 150
Machine 400
Total Payments 630 660 1,110
Net cash flow 330 340 282
Opening balance 40 370 710
Closing balance 370 710 992

(ii) Calculation of current ratio:


Inventory at the end of the three-month period:
This will be the finished goods for April sales of 1,500 units, which can be assumed to
be valued at the cost of production of Rs.400 per unit for materials and Rs.100 per unit
for variable overheads and wages. The value of the inventory is therefore
1,500 x 500 = Rs. 750,000.
Trade receivables at the end of the three-month period:
These will be March sales of Rs. 1,176,000.
Cash balance at the end of the three-month period:
This was forecast to be Rs. 992,000.
Trade payables at the end of the three-month period:
This will be the cash owed for March raw materials of Rs. 600,000.
Forecast current ratio
Assuming that current liabilities consist of trade payables alone:
Current ratio = (750,000 + 1,176,000 + 992,000)/600,000 = 4·9 times

b) Evaluation of decision to extend credit period:


Existing Receivable Turnover= 360/45 =8 times
New Receivable Turnover = 360/75 = 4.8 times
New Sales = Rs. 60m×1.15 = Rs. 69 m
Additional Sales = Rs. 69m –Rs. 60m = Rs. 9 m
Contribution on Estimated additional Sales = 90,00,000×20/100=Rs. 18,00,000
Addl. Receivables associated with increased sales = 90,00,000/4.8 = Rs.
18,75,000
Investment in additional receivables associated with new sales = 18,75,000×80/100
= Rs.15,00,000
Receivables on present sales, on existing terms = 600,00,000/8 =75,00,000
Receivables of present sales, on new terms = 600,00,000/4.8 = 125,00,000
Investment in additional receivables associated with original sales
= 125,00,000-75,00,000 = 50,00,000
Total Investment in additional Receivables = 1500,000+50,00,000=65,00,000
Or
= 15,00,000+50,00,000*80%
= 15,00,000+40,00,000
= 55,00,000*20%
= 11,000
Required Return (before tax) on Addl. Investments = 65,00,000×20/100=
Rs.13,00,000

Analysis: The return on increased sales is more than the interest cost on additional
investments in debtors balances. Hence, it is recommended to extend credit period.
4.
a) X Co. has made plans for the next year. It is estimated that the company will
employ total assets of Rs. 8,00,000; 50 percent of the assets being financed
by borrowed capital at an interest cost of 8 percent per year. The direct costs
for the year are estimated at Rs. 4,80,000 and all other operating expenses
are estimated at Rs. 80,000. The goods will be sold to customer at 150
percent of the direct costs. Tax rate is 25 percent.
Required: Calculate: 8
i) net profit margin
ii) return on assets
iii) assets turnover
iv) return on owners' equity

b) Omni Corporation has target capital structure of 60% equity and 40% debt.
The schedule of financing cost for Omni is shown below:
Amount of New After tax Cost of Amount of New Cost of Equity
Debt (Rs. Million) Debt Equity (Rs. Million)
0-99 4.2 % 0-199 6.5%
100-199 4.6% 200-399 8.0%
200-299 5% 400-599 9.5%

Required: 7
Calculate the Break Point for Omni Corporation and also calculate WACC
for alternate level of financing in those break points.
Answer:
a) Calculation of Net profit
Particulars Amount ( Rs.)
Sales (150% of Rs. 4,80,000) 7,20,000
Direct Costs 4,80,000
Gross profit 2,40,000
Operating Expenses 80,000
Profit before interest and tax 1,60,000
Interest charges (8% of Rs.4,00,000) 32,000
Profit before tax 1,28,000
Taxes @ 25% 32,000
Net profit after tax 96,000
(i) Net profit Margin = Profit after tax/Sales=96,000/7,20,000= 13.33%
(ii) Return on Assets = (EBIT -Tax paid)/Assets= (1,60,000-32,000)/8,00,000=16% Or
EBIT/Total Assets = 20% Or PAT/Total Assets = 12%
(iii) Assets Turnover= Sales/Assets=7,20,000/8,00,000=0.9 times
(iv) Return on Equity=Net profit after tax/Equity=96,000/4,00,000=24%

b) Omni will have a break point each time a component cost of capital changes, for a total of
four break points;
Break Point debt>100mn = Rs. 100 million/0.4 = Rs. 250 Million
Break Point debt>200mn = Rs. 200 million/0.4 = Rs. 500 Million
Break Point equity>200mn = Rs. 200 million/0.6 = Rs. 333 Million
Break Point equity>400mn = Rs. 400 million/0.6 = Rs. 667 Million
Omni Corporation‘s WACC for the different break points.
(i) [3
WACC for Alternative level of Financing at break points
Capital (Rs. Million) Equity Cost of Debt (40%) Cost of WACC
(60%) Equity Debt
250 [ 250- 332] 150 6.5% 100 4.6% 5.74%
333 [ 333-499] 200 8.0% 133 4.6% 6.64%
500 [500-666] 300 8.0% 200 5.0% 6.80%
667 [ 667 and Above] 400 9.5% 267 5.0% 7.70%
(ii) [4
[7
5.
a) Mathura Corporation has two different bonds currently outstanding. Bond M
has a face value of Rs. 20,000 and matures in 20 years. The bond makes no
payments for the first six years, then pays Rs. 1,200 every six months over
the subsequent eight years, and finally pays Rs. 1,500 every six months over
the last six years. Bond N also has a face value of Rs. 20,000 and a maturity
of 20 years; it makes no coupon payments over the life of the bond. The
required return on both of these bonds is 10 percent compounded
semiannually.
Required: 5
What is the current price of Bond M and Bond N?
You can use the following statistical figures:
PVIFA5%, 12 Years =8.8633 PVIFA5%, 28 Years =14.8981
PVIFA5%, 40 Years =17.1591 PVIFA5%, 40th Years=0.1420

b) The net sales of A Ltd. is Rs. 30 crores. Earnings before interest and tax of
the company as a percentage of net sales are 12%. The capital employed
comprises Rs. 10 crores of Equity Shares, Rs. 2 crores of 13% Cumulative
Preference Share and 15% Debentures of Rs. 6 crores. Income tax rate is
40%.
Required: 5
Calculate the return on equity for the company and show segment
decomposition of ROE due to the presence of Preference Share Capital and
Debentures.

c) Mr. X, an investor, is seeking the price to pay for the security whose
standard deviation is 5%. The correlation coefficient for the security with
the market is 0.75 and the market standard deviation is 4%. The return from
the risk-free securities is 6% and from the market portfolio is 11%. Mr. X
knows that only by calculating the required rate of return, he can determine
the price to pay for the security.
Required: 5
What is the required rate of return on the security?
Answer:
a) Bond M is redeemable bond and the value of redeemable bond is discounted
present value of Interest and Principal amount over the life of the bond.
Therefore the value of Bond M may be calculated as below:
Value of Bond M = PV of Interest for First 6 Years + PV of Interest for next 8 Years + PV of
Interest for last 6 Years + PV of Redemption Value of Principal
= 0+ 1,200×[PVIFA 5%, 28 Years –PVIFA 5%, 12 Years] + 1,500×[PVIFA 5%, 40 Years –
PVIFA 5%, 28 Years] + 20,000×[PVIF 5%, 40th Year]
= 0 + 1,200×[14.8981-8.8633]+ 1,500×[17.1591-14.8981]+20000×0.1420
= 7,241.76 + 3,391.50 + 2,840
= Rs. 13,473.26
Bond N is Zero Coupon Bond and the value of Zero Coupon bond is discounted present value of
Principal amount redeemed over the life of the bond. Therefore the value of Bond N may be
calculated as below:
Value of Bond N = PV of Redemption Value of Principal
= 20,000 × [PVIF5%, 40Years]
= 20,000 × 0.1420
= Rs. 2,840
b)
Calculation of Return on Equity [ROE]
[Figures in Rs. Crore]
Sales 30
Earnings Before Interest and Tax [12% of Sales] 3.6
Less: Interest [15% on NRs 6 Crore] 0.9
Earning Before Tax 2.7
Less: Tax@ 40% 1.08
Earning After Tax 1.62
Less: Preference Dividend 0.26
Earning for Equity Holders 1.36
Return on Equity ( 1.36/30) 13.60%
[0.5
Capital Employed = Equity Share Capital + Preference Share Capital + Debt
= 10+2+6
= Rs.18 Crore
Post Tax Return on Investment = [EBIT×(1-Tax)/Capital Employed]
= [3.6×(1-0.4)/18]
= 12%
Segment Decomposition of ROE may be analyzed as below;
ROE = Post Tax ROI + [Post Tax ROI- Cost of Preference Share]×[Preference Share/Equity] + [Post
Tax ROI- Post Tax Cost of Debt]×[Debt/Equity]
= 12% + [12%-13%]×[2/10]+[12%-9%]×[6/10]
=12% -0.20% + 1.8%%
=13.60%
The negative 0.2% and Positive 1.8% is the segment of ROE caused by presence of Preference Share
Capital and Debenture in the Capital Structure.
[1.5
Post Tax Cost of Debt = Coupon Rate×[1-Tax Rate]
= 15%×[1-0.40]
=9%

c) Standard deviation of the security = 5%


Correlation coefficient of portfolio with market = 0.75%
Market standard deviation = 4%
Risk-free rate of return = 6%
Expected return on market portfolio = 11%

The market sensitivity index i.e. the beta factor can be calculated as follows:

Standard deviation of the security 0.05


β= ------------------------------------------------------× CORsm = -------------×.75= 0.9375
Standard deviation of Market 0.04

Now, the expected return on the investment can be ascertained with the help of CAPM
equation as follows:
Rs = Irf +(Rm-Irf) β
= 6 + (11-6)×0.9375
= 10.69%

6. Write short note/ answer on: (4×2.5=10)


a) Effect of leverage on Capital Turnover and Working Capital Ratio
b) Credit Rating
c) Sensitivity Analysis in capital budgeting
d) Agency cost of equity and debt
Answer:
a) Effect of leverage on capital turnover and working capital ratio
An increase in sales improves the net profit ratio, raising the Return on Investment (R.O.I)
to a higher level. This however, is not possible in all situations; a rise in capital turnover is
to be supported by adequate capital base. Thus, as capital turnover ratio increases, working
capital ratio deteriorate, thus, management cannot increase its capital turnover ratio beyond
a certain limit. The main reasons for a fall in ratios showing the working capital position
due to increase in turnover ratios is that as the activity increases without a corresponding
rise in working capital, the working capital position becomes tight. As the sales increases,
both current assets and current liabilities also increase but not in proportion to current ratio.
If current ratio and acid test ratio are high, it is apparent that the capital turnover ratio can
be increased without any problem. However, it may be very risky to increase capital
turnover ratio when, the working capital position is not satisfactory.

b) Credit Rating
Credit rating essentially reflects the probability of timely repayment of principal and
interest by a borrower company. It indicates the risk involved in a debt instrument as well
its qualities. Higher the credit rating, greater is the probability that the borrower will make
timely payment of principal and interest and vice-versa.
It has assumed an important place in the modern and developed financial markets. It is a
boon to the companies as well as investors. It facilitates the company in raising funds in the
capital market and helps the investor to select their risk-return trade-off. By indicating
credit-worthiness of a borrower, it helps the investor in arriving at a correct and rational
decision about making investments. Credit rating system plays a vital role in investor
protection. Fair and good credit ratings motivate the public to invest their savings.
As a fee-based financial advisory service, credit rating is obviously extremely useful to the
investors, the corporate (borrowers) and banks and financial institutions. To the investors, it
is an indicator expressing the underlying credit quality of a security to be floated for in the
market. The investor is fully informed about the company as any effect of changes in
business/economic conditions on the company is evaluated and published regularly by the
rating agencies. The Corporate borrowers can raise funds at a cheaper rate with good rating.
It minimizes the role of the 'name recognition' and less known companies can also
approach the market on the basis of their rating.
c) Sensitivity Analysis
The net present value or Internal Rate of Return of a project is determined by analyzing the
after tax cash flows arrived at by combining forecasts of various variables like Sales
volume, unit selling price, unit variable cost, fixed cost etc. It is difficult to arrive at an
accurate and unbiased forecast of each variable. It can't be certain about the outcome of any
of these variables. The reliability of the NPV or IRR of the project will depend on the
reliability of the forecasts of variables underlying the estimates of net cash flows. To
determine the reliability of the project's NPV or IRR, we can work out how much
difference it makes if any of these forecasts go wrong. We can change each of the forecasts,
one at a time, to at least three values: Pessimistic, expected and optimistic. The NPV of a
project is recalculated under these different assumptions. The method of recalculating NPV
or IRR by changing each forecast is called Sensitivity Analysis.

Sensitivity Analysis is a way of analyzing change in the project's NPV or IRR for a given
change in one of the variables. It indicates how sensitive a project's NPV or IRR is to
changes in particular variables. It basically examines the sensitivity of the variables
underlying the computation of NPV or IRR rather than attempting to quantify risk. It can be
applied to any variable which is an input for the after tax cash flows. It can be conducted
with regard to volume, price, costs etc.

d) Agency cost of equity and debt


Agency cost refers to the cost incurred by a firm because of the problems associated with
the different interests of management and shareholder and the information asymmetry that
exists between the principal (shareholders) and the agent (management).
 Agency Cost of Equity
The agency cost of equity arises because of the difference in interests between the
shareholders and the management. As long as the management‘s interests diverge from
that of the shareholders, the shareholders will have to bear this cost. Management may
be tempted to take suboptimal decisions that may not work towards maximizing the
value for the firm. Any measures implemented to oversee and prevent this will have a
cost associated with it. So, the agency costs will include both, the cost due to the
suboptimal decision, and the cost incurred in monitoring the management to prevent
them from taking these decisions.
 Agency Cost of Debt
The agency cost of debt arises because of different interests of shareholders and debt-
holders. Assume that the management is in favor of the shareholders. If so, the
management can in many ways transfer the wealth to the shareholders and leaving debt-
holders empty handed. Anticipating such activities, the debt-holders will take various
preventive measures to disallow management from doing so. The debt holders may do
so in the form of higher interest rates to protect themselves from the losses. Alternatively
they may impose restrictive covenants.

7. Distinguish between: (4×2.5=10)


a) Spin Off and Carve Out
b) Factoring and Bills Discounting
c) Bank Overdraft and Clean Overdraft
d) Euro Bond and Foreign Bond
Answer:
a) Spin off and Carve out
A spin-off and carve-out are different methods that a company can use to divest certain
assets, a division or a subsidiary. While the choice of a specific method by the parent
company depends on a number of factors, the ultimate objective is to increase shareholder
value.
In a spin-off, the parent company distributes shares of the subsidiary that is being spun-off
to its existing shareholders on a pro rata basis, in the form of a special dividend. The parent
company typically receives no cash consideration for the spin-off. Existing shareholders
benefit by now holding shares of two separate companies after the spin-off instead of one.
The spin-off is a distinct entity from the parent company and has its own management. The
parent company may spin off 100% of the shares in its subsidiary, or it may spin off
controlling interest to its shareholders and hold a minority interest in the subsidiary.
In a carve-out, the parent company sells some or all of the shares in its subsidiary to the
public through an initial public offering (IPO). Unlike a spin-off, the parent company
generally receives a cash inflow through a carve-out. Since shares are sold to the public, a
carve-out also establishes a net set of shareholders in the subsidiary. A carve-out often
precedes the full spin-off of the subsidiary to the parent company's shareholders.

b) Factoring and Bills Discounting


The differences between Factoring and Bills discounting are as below:
 Factoring is called as "Invoice Factoring" where as Bills discounting is known as
"Invoice Discounting"
 In Factoring, the parties are known as client, factor and debtor where as in Bills
discounting, they are known as drawer, drawee and payee
 Factoring is a sort of management of book debts whereas bills discounting is a sort
of borrowing from commercial banks
 For factoring there is no specific law whereas in case of bills discounting the
negotiable instruments law is applicable
c) Bank Overdraft and Clean Overdraft
Bank Overdraft
Bank overdraft refers to an arrangement whereby the bank allows the customers to
overdraw from the current deposit account within a specified limit. The overdraft facility
is granted against the securities of assets or personal security as in case of cash credit.
Interest is charged only on the amount actually withdrawn (i.e. debit balance) for the
actual period of use (i.e., for the period the debit balance in current deposit account
remains outstanding). The cost of raising finance by this method is the interest charged
by the bank.

Clean Overdraft
Bank may entertain clean advances from those customers, which are financially, sound
and reputed for their integrity. The banks in this case rely upon the personal security of
the borrower. Banks are responsible for ensuring customer‘s credit worthiness before
providing them with clean overdraft as there is no assets securing the amount of
advance. The banks normally take guarantee from the persons whom they believe to be
credit worthy.

d) Euro Bond and Foreign Bond


A Eurobond is denominated in a currency other than the home currency of the country or
market in which it is issued. These bonds are frequently grouped together by the
currency in which they are denominated, such as eurodollar or euroyen bonds. Issuance
is usually handled by an international syndicate of financial institutions on behalf of the
borrower, one of which may underwrite the bond, thus guaranteeing purchase of the
entire issue. Eurobonds are issued outside the restrictions that apply to domestic
offerings. The earliest eurobonds were physically delivered to investors. As of 2016,
they are issued electronically through a range of services, including the Depository Trust
Company (DTC) in the United States and the Certificate less Registry for Electronic
Share Transfer (CREST) in the United Kingdom. Eurobonds are usually issued in bearer
form, which makes it easier for investors to avoid regulations and taxes.
Foreign bonds, unlike Eurobonds, are issued in a single country and are usually
denominated in that country‘s currency. Often, the country in which these bonds are
issued will draw distinctions between them and bonds issued by domestic issuers—
including different tax laws, restrictions on the amount issued, and tougher disclosure
rules. Foreign bonds often are nicknamed for the country where they are issued Yankee
bonds (United States), Samurai bonds (Japan), Rembrandt bonds (the Netherlands),
Bulldog bonds (Britain). Partly because of tougher regulations and disclosure
requirements, the foreign bond market hasn‘t grown in past years with the vigor of the
Eurobond market.
Cost and Management Accounting
Maximum Marks - 100
Total No. of Questions: 6 Total No. of Printed Pages - 14
Time Allowed - 3 Hours
Marks
All questions are compulsory. Working notes should form part of the answer.
Make assumptions wherever necessary.
1. The Management Team of Exe Ltd. is considering the possibility of undertaking a
single production process which jointly produces four products in standard
proportions. The output from each 10 kg. batch of raw material input in the
process, together with net realizable value per kg. of output immediately after the
split-off point, is as under:
Material Output per 10 kg. input Net realizable value per kg. of output (Rs.)
A 4 8
B 3 4
C 2 10
D 1 2
The costs of processing each 10 kg. input batch are Rs. 12 and the cost of the raw
material input is Rs. 4 per kg.
For each of the four material jointly produced there is the possibility of further
processing before sale. The further processing will entail manual operation and
mechanical processing as well as incurring some costs directly attributable to each
product. Details of the resources used in, and cost incurred by, the further
processing as well as the final price per kg. as under:
Material Machine hours Labour hours Other direct cost Sales price
A 2 1 4 17
B 6 1 2 13
C 4 5 3 36
D 2 2 2 9
―Other direct costs‖ are variable costs but exclude the cost of labour, also a
variable cost, at Rs. 3 per labour hour. Apart from ―other direct costs‖ and labour
costs , all other costs of this further processing are fixed and are expected amount
to Rs. 3,40,000 per annum.
Exe Ltd. has the opportunity to process 1,00,000 kg. of the basic raw material per
year and machine capacity is capable of fully processing this amount.
The Managing Director feels that all products which are subjected to further
processing must be treated as joint products and all products to be sold
immediately after the split-off point without further processing are to be treated as
by-products of the original process. The net costs of the joint process are allocated
to the joint products in the proportion to the contribution of each product line,
after considering the marginal costs after the split-off point and the sales revenues.
However, the Managing Director is uncertain whether the Rs. 3,40,000 fixed
production costs of the further processing should be allocated to products in
accordance with machine or labour hours.
Required: (6+14=20)
a) Specify which of the jointly produced materials should be subject to further
processing if the joint process is carried out.
b) Produce a product profitability report for the joint products, utilizing the
Managing Director‘s approach to the determination of joint and by-products,
for each of the methods of allocating fixed production overhead he has
mentioned. You may assume all the production will be sold.

Answers:
a) Comparative statement of contribution per kg from further processing and net
realizable value per kg at the split-off point
Materials
A B D
(Rs.) (Rs.) C (Rs.) (Rs.)
Labour cost 3 3 15 6
Other direct cost 4 2 3 2
Marginal cost 7 5 18 8
Sale price 17 13 36 9
Contribution 10 8 18 1
Net realizable value per kg at the split-off point 8 4 10 2
Advantage (Disadvantage) of further processing 2 4 8 -1
The above statement shows that the further processing of materials A, B and C is beneficial.
Hence, they should be processed further. However, material D is giving loss as a result of further
processing. Hence, it should be sold out art spilt- off point.
b) Joint Products profitability Report
i. Statement of profitability when fixed production cost of further processing are
apportioned to product according to machine hours
Products
A B C Total
Output in kg 40,000 30,000 20,000
Rs. Rs. Rs. Rs.
Sales 6,80,000 3,90,000 7,20,000 17,90,000
Costs:
Joint costs (Working Notes 1 & 2) 2,00,000 1,20,000 1,80,000 5,00,000
Labour costs 1,20,000 90,000 3,00,000 5,10,000
Other direct costs 1,60,000 60,000 60,000 2,80,000
Fixed overheads (Working Note 3) 80,000 1,80,000 80,000 3,40,000
Total costs 5,60,000 4,50,000 6,20,000 16,30,000
Profit(loss) 1,20,000 -60,000 1,00,000 1,60,000
ii. .Statement of profitability when fixed production cost of further processing are
apportioned according to labour hours
Products
A B C Total
Rs. Rs. Rs. Rs.
Sales 6,80,000 3,90,000 7,20,000 17,90,000
Costs:
Joint costs (Working Notes 1 & 2) 2,00,000 1,20,000 1,80,000 5,00,000
Labour costs 1,20,000 90,000 3,00,000 5,10,000
Other direct costs 1,60,000 60,000 60,000 2,80,000
Fixed overheads (Working Note 3) 80,000 60,000 2,00,000 3,40,000
Total costs 5,60,000 3,30,000 7,40,000 16,30,000
Profit(loss) 1,20,000 60,000 -20,000 1,60,000
Working Notes:
1) Computation of Joint Costs: Rs.
Raw Material Input (1,00,000 * Rs.4) 4,00,000
Processing Costs (1,00,000 *Rs. 12/10) 1,20,000
5,20,000
Less: Net realizable value of by-product D (1/10 * 1,00,000 *Rs.2) 20,000
Joint costs 5,00,000
2) Apportionment of Joint Costs in proportion to the contribution of each product
Total Allocated Joint
Product Output Contribution per kg Total Contribution Costs
kg Rs. Rs. Rs.
A 40,000 10 4,00,000 2,00,000
B 30,000 8 2,40,000 1,20,000
C 20,000 18 3,60,000 1,80,000
10,00,000 5,00,000
3) Computation of overhead recovery rates on machine / labour hour basis
Total Output Machine Hours Labour hours
Product Per kg Total Per kg Total
A 40,000 2 80,000 1 40,000
B 30,000 6 1,80,000 1 30,000
C 20,000 4 80,000 5 1,00,000
3,40,000 1,70,000
Overhead Recovery Rates:
Per machine hour: Rs, 3,40,000/3,40,000 = Re. 1 per hour
Per labour hour: Rs. 3,40,000/1,70,000 = Rs, 2 per hour
2.
a) The Nepal Sofa Company (NS Co.) makes sofas. It has recently received a
request from a customer to provide a one-off order of sofas, in excess of
normal budgeted production. The order would need to be completed within
two weeks. The following cost estimate has already been prepared:
Additional Rs.
Information
Direct materials: (Notes)
Fabric 200 m² at Rs. 17 per m² 1 3,400
Wood 50 m2 at Rs. 8.20 per m² 2 410
Direct labour:
Skilled 200 hours at Rs. 16 per hour 3 3,200
Semi-skilled 300 hours at Rs. 12 per hour 4 3,600
Factory overheads 500 hours at Rs. 3 per hour 5 1,500
–––––
Total production cost 12,110
Administration overheads at 10% of total production cost 6 1,211
–––––
Total cost 13,321
–––––
Additional Information (Notes):
a. The fabric is regularly used by NS Co. There are currently 300 m² in
inventory, which cost Rs. 17 per m². The current purchase price of the
fabric is Rs. 17·50 per m².
b. This type of wood is regularly used by NS Co. and usually costs Rs. 8·20
per m². However, the company‘s current supplier‘s earliest delivery time
for the wood is in three weeks‘ time. An alternative supplier could deliver
immediately but they would charge Rs. 8·50 per m². NS Co. already has
500 m² in inventory but 480 m² of this is needed to complete other existing
orders in the next two weeks. The remaining 20 m² is not going to be
needed until four weeks‘ time.
c. The skilled labour force is employed under permanent contracts of
employment under which they must be paid for 40 hours‘ per week‘s
labour, even if their time is idle due to absence of orders. Their rate of pay
is Rs. 16 per hour, although any overtime is paid at time and a half. In the
next two weeks, there is spare capacity of 150 labour hours.
d. There is no spare capacity for semi-skilled workers. They are currently
paid Rs. 12 per hour or time and a half for overtime. However, a local
agency can provide additional semi-skilled workers for Rs. 14 per hour.
e. The Rs. 3 absorption rate is NS Co.‘s standard factory overhead absorption
rate; Rs. 1·50 per hour reflects the cost of the factory supervisor‘s salary
and the other Rs. 1·50 per hour reflects general factory costs. The
supervisor is paid an annual salary and is also paid Rs. 15 per hour for any
overtime he works. He will need to work 20 hours‘ overtime if this order is
accepted.
f. This is an apportionment of the general administration overheads incurred
by NS Co.
Required: (4+6=10)
i) Prepare, on a relevant cost basis, the lowest cost estimate which could be
used as the basis for the quotation.
ii) Explain briefly your reasons for including or excluding each of the costs in
your estimate.
b) The standard material cost for a mix of one tonne of final product is based on
the following:
Material Usage (kg.) Price per kg. (Rs.)
A 250 12
B 450 15
C 600 20
During the month of July, 2017, 12 tonnes of final product were produced
from the following:
Material Usage (tonnes) Cost (Rs.)
A 3.50 45,500
B 6.10 85,400
C 6.50 1,43,000
You are required to calculate the material variances and verify them. 10

Answers:
a)
Nepal Sofa Company
Cost Estimate for the quotation
Direct materials: Note Rs.
Fabric 200 m² at Rs. 17·50 per m² 1 3,500
Wood 20 m2 at Rs. 8·20 per m2 2 164
30 m2 at Rs. 8·50 per m2 2 255
Direct labour:
Skilled 50 hours at Rs. 24 per hour 3 1,200
Semi-skilled 300 hours at Rs. 14 per hour 4 4,200
Factory overheads 20 hours at Rs. 15 per hour 5 300
Administration overheads 6 –
––––––
Total cost 9,619
––––––

Reasons for the costs as included above:


1 Since the material is in regular use by NS Co, it is replacement cost which is
the relevant cost for the contract.
2 30 m will have to be ordered from the alternative supplier for immediate
delivery but the remaining 20 m can be used from inventory and replaced by
an order from the usual supplier at a cost of Rs. 8·20 per m.
3 There is no cost for the first 150 hours of labour because there is spare
capacity. The remaining 50 hours will be paid at time and a half, which is Rs.
16 x 1·5, i.e. Rs. 24 per hour.
4 NS Co will choose to use the agency workers, who will cost Rs. 14 per hour,
since this is cheaper than paying existing semi-skilled workers at Rs. 18 per
hour (Rs.12 x 1·5) to work overtime.
5 None of the general factory costs are incremental, so they have all been
excluded. However, the supervisor‘s overtime pay is incremental, so has been
included. The supervisor‘s normal salary, on the other hand, has been
excluded because it is not incremental.
6 These are general overheads and are not incremental, so no value should be
included for them.

b)
Working Notes
1. Total standard cost (TSC)= (SQ x SP)
Rs.
Material A: 250 x12 = 3000
Material B: 450 x 15 = 6750
Material C: 600 x 20 = 12000
TSC for one tonne of final output 21750
TSC for actual output of 12 tonnes = 21750 x 12= Rs.261000
2. Total Actual Cost (TAC) and Actual Price (AP):

Material AQ (kg) Cost (Rs.) AP=(Cost/AQ)


(tonnes x1000 kg) (per kg Rs.)

A 3500 45500 13
B 6100 85400 14
C 6500 143000 22

TAC 16100 273900

Rs.
3. AQ x SP : Material A: 3500 x 12 = 42000
Material B: 6100 x 15 = 91500
Material C: 6500 x 20 = 130000
= 263500
4. Revised Standard Quantity (RSQ) = Total Actual Quantity divide into standard mix ratio.
RSQ : for A= 16100 kg x 250/1300 = 3096 kg.
: for B= 16100 kg x 450/ 1300= 5573 kg.
: for C= 16100 kg x 600/ 1300= 7431 kg.

5. (RSQ x SP) : Material A = 3096 x 12 = 37152


: Material B = 5573 x 15 = 83595
: Material C = 7431 x 20 = 148620
269367
6. Standard yield (SY) by using actual Quantity one tonne SY from 250+450+600=1300 kg
SY from Actual Quantity = 16100/1300=12.3846 tonnes.

Calculation of Material Variances:

1. MCV = TSC –TAC or (SQ x SP) - (AQ x AP)


= Rs.261000 – Rs.273900 = Rs.12900 (A)
2. MPV = AQ (SP-AP) or (AQ x SP)-(AQ x AP)
=Rs.263500 – Rs.273900 = Rs.10400 (A)
3. MUV = SP (SQ - AQ) or (SP x SQ) - (SP x AQ)
= Rs.261000 – Rs.263500 = Rs.2500 (A)
4. Material Mix Variance (MMV)
= SP (RSQ-AQ) or (SP x RSQ) - (SP x AQ)
= Rs.269367 – Rs.263500 = Rs.5867 (F)
5. Material Sub Usage Variance (MSUV)(Yield)
= SP (SQ – RSQ) or (SP x SQ) – (SP x RSQ)
= Rs.261000 – Rs.269367 =Rs.8367 (A)

Verification:
(i) MCV = MPV + MUV
Rs.12900 (A) = Rs.10400 (A) + Rs.2500 (A)
OR
Rs.12900 (A) = Rs.12900 (A)
(ii) MUV = MMV + MSUV
Rs.2500 (A) = Rs.5867 (F) + Rs.8367 (A)
OR
Rs.2500 (A) = Rs.2500 (A)
3.
a) Following information have been extracted from the cost records of XYZ Pvt.
Ltd:
Rs.
Stores:
Opening balance 54,000
Purchases 2,88,000
Transfer from WIP 1,44,000
Issue to WIP 2,88,000
Issue for repairs 36,000
Deficiency found in stock 10,800

Work-in-progress:
Opening balance 1,08,000
Direct wages applied 1,08,000
Overheads charged 4,32,000
Closing balance 72,000

Finished Production:
Entire production is sold at a profit of 15% on cost at WIP
WIP
Wages paid 1,26,000
Overheads incurred 4,50,000
Draw the Stores Ledger Control Account, Work-in-Progress Control Account,
Overheads Control Account and Costing Profit and Loss Account. 8
b) Compute the machine hour rate from the following data: 8
(Rs.)
Cost of machine 1,00,000
Installation Charge 10,000
Estimated scrap value after the expiry of its life (15 yrs.) 5,000
Rent and rates for the shop per month 200
General lighting for the shop per month 300
Insurance premium for the machine per annum 960
Repair and maintenance expenses per annum 1,000
Power consumption - 10 units per hour
Rates of power per 100 units 20
Estimated working hours per annum - 2,200. This includes setting-up
time of 200 hours
Shop supervisor's salary per month 600
The machine occupies ¼ of the total area of the shop. The supervisor is
expected to devote 1/5 of his time for supervising the machine.
c) What is the limitation of Break-even chart? 4
Answers:
a)
Stores Ledger Control A/c

Particulars Rs. Particulars Rs.


To Balance b/d 54,000 By Work in Process A/c 2,88,000
To General Ledger 2,88,000 By Overhead Control A/c 36,000

Adjustment A/c By Overhead Control A/c 10,800*


To Work in Process 1,44,000 (Deficiency)
A/c By Balance c/d 1,51,200

4,86,000 4,86,000
*Deficiency assumed as normal (alternatively can be treated as abnormal loss)
Work in Progress Control A/c
Particulars Rs. Particulars Rs.
To Balance b/d 1,08,000 By Stores Ledger Control 1,44,000
To Stores Ledger Control A/c 2,88,000 a/c Costing P/L A/c
By 7,20,000
To Wages Control A/c 1,08,000 (Balancing figures being
To Overheads Control a/c 4,32,000 of finished goods)
Cost
By Balance c/d 72,000
9,36,000 9,36,000
Overheads Control A/c
Particulars Rs. Particulars Rs.
To Stores Ledger Control A/c 36,000B By
ByBy
Work in Process A/C 4,32,000
To Stores Ledger Control A/c 10,800 By Balance c/d 82,800
To Wages Control A/c 18,000 (Under absorption)
(Rs.1,26,000- Rs.1,08,000)
To Gen. Ledger Adjust. A/c 4,50,000
5,14,800 5,14,800
Costing Profit & Loss A/c
Particulars Rs. Particulars Rs.
To Work in progress 7,20,000 By Gen. Ledger Adjust. A/c 8,28,000
To Gen. Ledger Adjust. 1,08,000 (Sales) (Rs. 7,20,000 × 115%)
(Profit)
A/c
8,28,000 8,28,000
Note: Overhead control A/C under absorption can be transferred to costing PL A/C.
b)
Computation of Machine Hour Rate
Particulars: Rs. Rs.
Standing charges:
Rent and rates (200 *12) * ¼ 600
General lighting (300*12)*1/4 900
Insurance Premium 960
Shop supervisor's salary (600*12) *1/5 1440
Depn (1,10,000 - 5,000)/15 7000
10,900
Hourly rate for standing charges ( Rs.10900/ 2,000hrs) 5.45
Machine Expenses:
Repairs and Maintenance (1,000/2,000) 0.50
Power -10 units per hour @ Rs.0.20 per unit 2.00
Machine - hour rate 7.95
Note: Setting- up time has been presumed as non- productive time and hence
productive time is only 2,000 Hours.

c)
Break even analysis is fundamentally a static analysis as it assumes almost everything
constant (e.g., constant total fixed costs, variable cost per unit, selling price,
productivity, sales mix in case of multi products etc.) The limitations which make the
assumptions to be unrealistic are given below:
1. All costs cannot be separated into fixed and variable components with
accuracy.
2. Fixed costs may change because of change in management policy or after
a range of activity.
3. Variable cost per unit may change because of operation of law of
increasing returns or decreasing returns.
4. Selling price may change because of increase or decrease in output, market
demand & supply, competition etc.
5. In case of multiple products, the sales mix need not necessarily be
constant.
6. In case of multiple products, separate break even points are to be
calculated. This poses a problem of apportionment of fixed costs to each
product.
7. Entire production need not necessarily be sold in practice.
8. When a number of products are produced separate break-even chart will
have to be calculated. This poses a problem of apportionment of fixed
expenses to each product.
9. Break-even charts ignore the capital employed in business which is one of
the important guiding factors in the determination of profitability.

4.
a) XYZ Company produces three products A, B and C. Selling price of product
A Rs. 100, Product B Rs. 80 and Product C Rs. 50.Variable cost per unit of A,
B and C product are Rs. 50, Rs. 40 and Rs. 20 respectively. The product and
sale mixed of A, B and C product are 20%, 30% and 50% respectively. The
total fixed costs are Rs. 14,80,000.
Considering the above information, you are required to find out overall break-
even quantity and product wise break-up of such quantity. 5
b) Juntara Construction Ltd. undertook a contract for Rs. 5,00,000 on 1st July,
2015. On 30th June 2016 when the accounts were closed, the following details
about the contract were gathered:
Particulars Rs.
Materials purchased 1,00,000
Wages paid 45,000
General expenses 10,000
Plant Purchased 50,000
Materials on hand 30-6-2016 25,000
Wages accrued 30-6-2016 5,000
Work certified 2,00,000
Cash received 1,50,000
Depreciation of plant 5,000
Work uncertified 15,000
The above contract contained escalation clauses which read as follows:
―In the event of prices of materials and rates of wages increase by more than
5% the contract price would be increased accordingly by 25% of the rise in the
cost of materials and wages beyond 5% in each case‖.
It was found that since the date of signing the agreement the prices of
materials and wage rates increased by 25%. The value of the work certified
does not take into account the effect of the above clause.
Prepare the contract account. 7
c) Explain the methods of valuation of work-in-process. 3
Answers:
a)
Product A B C
Selling price per Unit 100 80 50
Variable cost per Unit 50 40 20
Contribution per Unit 50 40 30
Manufacture and sold unit in % 20 30 50
Composite contribution
= (50×0.20+40×0.30+30×0.5)
=10+12+15
= Rs 37 per unit
Overall Break even quantity= Fixed cost
Contribution
=14,80,000
37
= 40,000 Units
The product wise break-up of this quantity is as follows:
A 40,000 20% 8,000 Units
B 40,000 30% 12,000 Units
C 40,000 50% 20,000 Units

b) In the books of Juntara construction Ltd.


Contract Account
Dr. for the period 1st July 2015 to 30th June 2016
Cr.
Particulars Amount Particulars Amount
To, Material purchased A/c 1,00,000 By, Material on hand 25,000
To, Wages A/c 50,000 By, Work certified 2,00,000
To, General Expenses A/c 10,000 By, Work uncertified 15,000 2,15,000
To, Depreciation on plant 5,000 By, Escalation claim 5,000
To, Notional Profit c/d 80,000

2,45,000 2,45,000
To, P & L A/c 20,000 By, Notional Profit b/d 80,000
To, Reserve c/d 60,000
80,000 80,000
Working Notes:
1. Contract Escalation
Cost of material & wages incurred = Rs. (1,00,000 + 45,000 + 5,000 – 25,000 )
= Rs.1,25,000
Cost of material & wages before increase in prices = Rs. (1,25,000 x 100/125 )
= Rs.1,00,000
Increase in contract price = 25/100 [Rs.1,25,000 – (Rs.1,00,000 x 105/100)]
= Rs.5,000
2. Percentage of completion of Contract
Work Certified = 2,00,000 = 40%
× 100
Contract Price 5,00,000

3. Profit to be transferred to P&L Account


1/3 x Notional Profit x Cash Received/Work Certified
=1/3 x Rs.80,000 x Rs.1,50,000/Rs.2,00,000
=Rs. 20,000
c) The valuation of work-in-process can be made in the following three ways, depending
upon the assumptions made regarding the flow of costs.
First-in-first out (FIFO) method
Last-in-first out (LIFO) method
Average cost method
A brief account of the procedure followed for the valuation of work-in-process under
the above three methods is as follows;
FIFO method:
According to this method the units first entering the process are completed first. Thus
the units completed during a period would consist partly of the units which were
incomplete at the beginning of the period and partly of the units introduced during the
period. The cost of completed units is affected by the value of the opening inventory,
which is based on the cost of the previous period. The closing inventory of work-in-
process is valued at its current cost.
LIFO method:
According to this method units last entering the process are to be completed first. The
completed units will be shown at their current cost and the closing-work in process
will continue to appear at the cost of the opening inventory of work-in-progress along
with current cost of work in progress if any.
Average cost method:
According to this method opening inventory of work-in-process and its costs are
merged with the production and cost of the current period, respectively. An average
cost per unit is determined by dividing the total cost by the total equivalent units, to
ascertain the value of the units completed and units in process
5.
a) In a manufacturing concern bonus to workers is paid on a slab rate based on
cost savings towards labour and overheads. The following are the slab rates:
Up to 10% saving - 5% of the earning
Up to 15% saving - 9% of the earning
Up to 20% saving - 13% of the earning
Up to 30% saving - 21% of the earning
Up to 40% saving - 28% of the earning
Above 40% saving - 32% of the earning
The wage rate per hour of workers - P, Q, R and S are respectively Rs. 10, Rs.
11, Rs. 12 and Rs. 14. Overheads are recovered on direct wages at the rate of
200%. Standard cost under wages and overhead per unit of production is fixed
at Rs. 300. The workers have completed one unit each in 8, 7, 5½ and 5 hours
respectively.
Calculate in respect of each worker: (3+2+2=7)
i) Amount of bonus earned
ii) Total earnings
iii) Total earnings per hour
b) Outline the types of budget commonly used. 4
c) What is the practical application of differential cost analysis? 4
Answers:
a)
Statement showing computation of amount of bonus, total earnings and earnings per hour
Rs.
Particulars P Q R S
1. Standard cost of wages and overheads 300 300 300 300
2. Time taken in hours 8 7 5.5 5
3. Rate per hour 10 11 12 14
4. Actual wages (2 x 3) 80 77 66 70
5. Overheads (200% of wages) 160 154 132 140
6. Actual cost of Labour & Overhead (4 + 5) 240 231 198 210
7. Savings (1 – 6) 60 69 102 90
8. % of savings (7/1 x 100) 20% 23% 34% 30%
9. % of bonus applicable 13% 21% 28% 21%
10. a) Amount of bonus earned (4 x 9) 10.40 16.17 18.48 14.70
11. b) Total earnings (4 + 10) 90.40 93.17 84.48 84.70
12. c) Total earnings per hour (11/2) 11.30 13.31 15.36 16.94

b) The Institute of Cost and Management Accountants (CIMA) defined budget as 'A
quantitative expression of a plan for a defined period of time. It may include planned
sales volumes and revenues, resource quantities, costs and expenses, assets, liabilities
and cash flows.' The types of budget commonly used are:
Master Budget
A master budget is an aggregate of a company's individual budgets designed to
present a complete picture of its financial activity and health. The master budget
combines factors like sales, operating expenses, assets, and income streams to allow
companies to establish goals and evaluate their overall performance, as well as that of
individual cost centers within the organization
Operating Budget
An operating budget is a forecast and analysis of projected income and expenses over
the course of a specified time period. To create an accurate picture, operating budgets
must account for factors such as sales, production, labor costs, materials costs,
overhead, manufacturing costs, and administrative expenses. Operating budgets are
generally created on a weekly, monthly, or yearly basis.
Cash Flow Budget
A cash flow budget is a means of projecting how and when cash comes in and flows
out of a business within a specified time period. It can be useful in helping a company
determine whether it's managing its cash wisely. Cash flow budgets consider factors
such as accounts payable and accounts receivable to assess whether a company has
ample cash on hand to continue operating, the extent to which it is using its cash
productively, and its likelihood of generating cash in the near future
Financial Budget
A financial budget presents a company's strategy for managing its assets, cash flow,
income, and expenses. A financial budget is used to establish a picture of a company's
financial health and present a comprehensive overview of its spending relative to
revenues from core operations.
c) Differential costing is a technique where mainly differential costs are considered
relevant. Differential cost is the difference in total costs between two acceptable
alternative courses of action. Differential cost analysis is usually made to facilitate
managerial decisions of following kind:
(a) Determination of the most profitable levels of production and price
(b) Acceptance of special orders – offer at a lower price or offering a quotation at
lower selling price in order to increase the capacity.
(c) Sell a product as it is or after further processing
(d) Determination of right price at which materials may be purchased
(e) Decisions regarding alternative capital investment and plant replacement
(f) Decisions such as changing the product mix, method of production, make or buy,
adding new product, etc.
6. Write short notes: (4×2.5=10)
a) Continuous costing
b) Valuation of material receipt
c) Classification of overhead by nature
d) Stock control account
Answers:
a) A category of costing methods applicable to the repetitive production processes with
continuous operations in which the products and services are identical and cannot be
segregated. It enables the management to know what it is costing to do the jobs in
hand and helps it to take corrective action in time to check wastages and
losses. Continuous costing involves very often the use of estimates of expenditure
instead of actual.
As will be seen later, actual expenditure on a job regarding materials and labour can
always be known easily but actual information about other expenses may not be
available for some-time. Hence an estimate of such expenses is necessary.

b) The invoice of material purchased from the market some time contain such items such
as trade discount, quantity discount, freight, duty, insurance, cost of packing, sale tax,
excise duty, cash discount etc. Under such a situation the general principal is that all
the cost incurred up to the point of procuring and storing materials should constitute
the cost of material purchase .The amount of trade discount. The amount of trade
discounts, quantity discounts and VAT are deducted from the invoice of materials
purchased. The transport charges, sale tax, insurance, cost of packing, customs and
excise duty should be included in the invoice cost of materials. The cash discount is
considered as financial gain, so it is kept outside the domain of material cost. In case
the containers are returnable, their resale value should also be taken in the invoice
price of material to correctly ascertain the cost of material purchased. The cost of
material purchased so determined may be used for the entry of material in the Store
Ledger.
c)
i) Fixed or constant: These are expenses that are not affected by any variation in the
volume of activity, e.g., managerial remuneration, rent, that part of depreciation
which is dependent purely on efflux of time, etc. Fixed or constant expenses
remain the same from from one period to another except when they are
deliberately changed, e.g., on increments being granted to staff or additional staff
being engaged.
ii) Variable: Expensed that change in proportion to the change in the volume of
activity; when output goes up by 10% the variable expenses also go up by 10%.
Correspondingly, on a decline of the output, these expenses also decline
proportionately e.g., power consumed; consumable stores; repairs and
maintenance and depreciation (on account of wear and tear) are dependent on the
use of assets.
Variable expenses are generally constant per unit of output or activity. Suppose
variable expenses amount to Rs. 10,000 for a production of 2,000 units i.e; Rs. 5
per unit. When output goes up to 2,200 units, with an increase of 10%, the
variable expenses amount to Rs. 11,000 i.e., 10,000 plus 10%, however, the cost
per unit will be the same as before.
iii) Semi variable: The expenses that either (a) do not change when there is a small
change in the level of activity but change whenever there is a slightly big change
and the change are in small steps; or (b) change in the same direction as change in
the level of activity but not in the same proportion. An expenses for example, may
not change if output goes up or come down by 5 % but may change by 3% when
there is an increase in production between 5% and 10%. Similarly, another item of
expenses may change by 1% for every 2% change in activity. Examples of such
expenses are: delivery van expenses, telephone charges, depreciation as a whole.
d) The account is prepared for each of the following cost items.
I. Raw Material: This account has opening stock and purchases on debit side and
material issues on credit side.
II.WIP: This account is debited with opening stock and factory overhead and credited
with cost of goods finished .The closing stock, if any, will be carried forward to the
next year.
III. Finished stock: This account is known as the finished goods account also. It is
debited with goods finished and credited with the cost of sales.
The above stock accounts are usually used in place of the stock and purchase account
which is maintained in the financial book.
Business Communication
Maximum Marks – 100
Total No. of Questions - 8 Total No. of Printed Pages -7
Time Allowed – 3 Hours
Marks
All questions are compulsory.
Section -'A'
1. Read the following case carefully and answer the questions given below: (4×5=20)
Effective Communication as a Motivator
One common complaint employees voice about supervisors is inconsistent
messages – meaning one supervisor tells them one thing and another tells them
something different. Imagine you are the supervisor/manager for each of the
employees described below. As you read their case, give consideration to how you
might help communicate with the employee to remedy the conflict.

Hari is a 27-year old who is a foodservice manager at a casual dining restaurant.


He is responsible for supervising and managing all employees in the back of the
house. Employees working in the back of the house range in age from 16 years old
to 55 years old. In addition, the employees come from diverse cultural and ethnic
backgrounds. For many, English is not their primary language.

Hari tries his best to keep up with food safety issues in the kitchen but he admits
it‘s not easy. Employees receive ―on the job training‖ about food safety basics.
But with high turnover of employees, training is often rushed and some new
employees are put right into the job without training if it is a busy day. Eventually,
most employees get some kind of food safety training. The owners of the
restaurant are supportive of Hari in his food safety efforts because they know if a
food safety outbreak were ever linked to their restaurant; it would likely put them
out of business. Still, the owners note there are additional costs for training and
making sure food is handled safely.

One day Hari comes to work and is rather upset even before he steps into the
restaurant. Things haven‘t been going well at home and he was lucky to rummage
through some of the dirty laundry and find a relatively clean outfit to wear for
work. He admits he needs a haircut and a good hand scrubbing, especially after
working on his car last evening. When he walks into the kitchen he notices several
trays of uncooked meat sitting out in the kitchen area. It appears these have been
sitting at room temperature for quite some time. Hari is frustrated and doesn‘t
know what to do. He feels like he is beating his head against a brick wall when it
comes to getting employees to practice food safety.

Hari has taken many efforts to get employees to be safe in how they handle food.
He has huge signs posted all over the kitchen with these words: KEEP HOT
FOOD HOT AND COLD FOOD COLD and WASH YOUR HANDS ALWAYS
AND OFTEN. All employees are given a thermometer when they start so that
they can temp food. Hand sinks, soap, and paper towels are available for
employees so that they are encouraged to wash their hands frequently.
Questions:
a) What are the communication challenges and barriers Hari faces?
b) What solutions might Hari consider in addressing each of these challenges and
barriers?
c) What Standard Operating Procedures (SOPs) would be helpful for Hari to
implement and enforce?
d) What are some ways Hari might use effective communication as a motivator
for employees to follow safe food handling practices?
Answer:
a) Communication challenges abound at any workplace. Hari has some common
challenges in his operation.
 Language barriers: not all employees speak English as their first language making
verbal communication a challenge at times.
 Generational (age) barriers: having employees in various age categories can pose a
unique set of challenges. While the younger generation is used to texting and using
shortened messaging, their vocabulary may not be consistent with that of older
employees. Work values and attitudes may also affect communication between
younger and older employees.
 Cultural and ethnic barriers: Cultural differences in food safety practices may be a
challenge for Hari to overcome.
 Non verbal challenges: Hari‘s body language (appearance) is telling others he does
not care about personal appearance and cleanliness.
 Emotional barriers: Emotional barriers can interfere with effective communication.
Hari comes into work after a rough start at home. These negative emotions are
affecting how he communicates with the employees.

b) Hari might consider the following solutions to the identified challenges and barriers:
 Language barriers: Several potential solutions might be addressed here including
posting signs in employees‘ primary language. Putting signage with visuals, not just
words. Hari might learn some simple words in the employees‘ primary language to
help show interest in the employees.
 Generational (age) barriers: Currently there are 4 generations in the workforce and
each potentially has a different preferred method of communication. While the
younger generation might prefer to receive text messages as their preferred way of
communication, older employees may not find this method of communication
acceptable. Consider your employee‘s preferences and be willing to communicate a
message in a few different ways.
 Cultural and ethnic barriers: Hari may need to identify cultural beliefs and work to
understand the ethnic barriers related to food safety. For instance, two employees
come from the same country and they have made comments that controlling
temperature in their country is not a priority; food can be at room temperature for long
periods of time and nothing ever happened.
 Non verbal challenges: Hari‘s appearance is a nonverbal cue to employees. His
appearance is important as he is a role-model to the employees. His actions and
behaviors should be consistent with what he is expecting of them. For example,
because he is expecting the employees to follow proper handwashing procedure, he
should also use proper handwashing procedures.
 Emotional barriers: Emotional barriers can interfere with effective communication. It
will be important for Hari to get his emotions ―in check‖ prior to starting work.
Having self-awareness and potentially seeking outside assistance may be possible
solutions here.

c) Standard Operating Procedures (SOPs) serve as the written documentation of best


practices and serves as the framework for organizational policy and structure. SOPs
identify the who, what, why, when, and how of foodservice practices for employees.
Part of the supervisor‘s role is to assure SOPs are in place, communicated to employees,
and followed. For this case, two helpful SOPs would be:
 New Employee Orientation SOP: Due to time constraints of ―busy days‖ and high
turnover, Hari currently finds orienting new employees to be a challenge. Although it
may take time at the beginning, the pay off may be great and save time in the long
run.
 Employee Health and Personal Hygiene SOP: Although SOPs are generally intended
for employees, it is important to note that as role models, supervisors/managers
should generally adhere to operational SOPs. Hari‘s own lack of adherence to
personal hygiene standards indirectly tells employees that appearance and hygiene are
not important. His own personal hygiene and appearance should serve as an example
to employees – clean, unwrinkled clothing, clean hands, free from grease and dirt with
neatly trimmed hair are a must for him to be a motivator for his employees to have
good appearance and hygiene. At present, his non verbal communication is telling
them personal hygiene and appearance is not important.

d) All supervisors, including Hari, have many ways to use effective communication in
motivating employees. A few suggestions are listed below:
 Provide sincere and encouraging words when employees follow safe food handling
behaviors. Use a communication method appropriate for an employee, so supervisor
must know a bit about the employee. For a high schooler, maybe it‘s a quick ―thank
you‖ text message or an older employee it might be a hand written thank you note.
 Serve as a role model through verbal and non verbal communication. It‘s said that
―actions‖ speak louder than words, so Hari can, through his actions, convey a
message to employees. For example: wear a clean uniform.
 Nonverbal (non-word) communication is the transmission of information in addition
to words in a communication to an audience or receiver of the communication content.

2. What are the advantages and disadvantages of working in teams, describe the
characteristics of effective teams. (6+4=10)
Answer:
A team is a unit of two or more people who share a mission and the responsibility for
working to achieve a common goal. Problem-solving teams and task forces assemble to
resolve specific issues and then disband when their goals have been accomplished. Such
teams are often cross-functional, pulling together people from a variety of departments
who have different areas of expertise and responsibility. The diversity of opinions and
experiences can lead to better decisions, but competing interests can lead to tensions that
highlight the need for effective communication. Committees are formal teams that usually
have a long-life span and can become a permanent part of the organizational structure.
Committees typically deal with regularly recurring tasks, such as an executive committee
that meets monthly to plan strategies and review results.

The teamwork interactions among the employees represent one of the most essential
elements of interpersonal communication. Collaboration —working together to meet
complex challenges—has become a core job responsibility of the professionals. It‘s a
virtual guarantee that everyone will be expected to collaborate in at least some of his/her
work activities. Communication skills will pay off handsomely in these interactions,
because the productivity and quality of collaborative efforts depend heavily on the
communication skills of the professionals involved.

Advantages of teams

When teams are successful, they can improve productivity, creativity, employee
involvement, and even job security. Teams are often at the core of participative
management, the effort to involve employees in the company‘s decision making. A
successful team can provide a number of advantages such as:

Increased information and knowledge: By pooling the experience of several


individuals, a team has access to more information in the decision-making process.

Increased diversity of views: Team members can bring a variety of perspectives to the
decision-making process—as long as these diverse viewpoints are guided by a shared
goal.

Increased acceptance of a solution: Those who participate in making a decision are


more likely to support it and encourage others to accept it.

Higher performance levels: Working in teams can unleash new levels of creativity and
energy in workers who share a sense of purpose and mutual accountability. Effective
teams can be better than top-performing individuals at solving complex problems.

Disadvantages of teams
Although teamwork has many advantages, it also has a number of potential
disadvantages. At the worst, working in teams can be a frustrating waste of time. Teams
need to be aware of and work to counter the following potential disadvantages:
Groupthink: Like other social structures, business teams can generate tremendous
pressures to conform with accepted norms of behaviour. Groupthink occurs when peer
pressures cause individual team members to withhold contrary or unpopular opinions.
The result can be decisions that are worse than the choices the team members might have
made individually.
Hidden agendas: Some team members may have a hidden agenda —private,
counterproductive motives, such as a desire to take control of the group, to undermine
someone else on the team, or to pursue a business goal that runs counter to the team‘s
mission.
Cost: Aligning schedules, arranging meetings, and coordinating individual parts of a
project can eat up a lot of time and money.

Characteristics of effective teams


The most effective teams have a clear objective and shared sense of purpose, have a
strong sense of trust, communicate openly and honestly, reach decisions by consensus,
think creatively, and know how to resolve conflict. Teams that have these attributes can
focus their time and energy on their work, without being disrupted by destructive conflict.
In contrast, teams that lack one or more of these attributes can get bogged down in
conflict or waste time and resources pursuing unclear goals. Two of the most common
reasons cited for unsuccessful teamwork are a lack of trust and poor communication. A
lack of trust can result from team members being suspicious of one another‘s motives or
ability to contribute. Communication breakdowns are most likely to occur when teams
operate across cultures, countries, or time zones. Thus, the group dynamics is necessary
among the team members. The interactions and processes that take place among the
members of a team are called group dynamics. Productive teams tend to develop clear
norms, informal standards of conduct that members share and that guide member
behavior. Group dynamics are influenced by several factors: the roles that team members
assume, the current phase of team development, the team‘s success in resolving conflict,
and the team‘s success in overcoming resistance.

3.
a) Write an e-mail responding to an announcement for a vacancy. 5
b) What are the supplementary parts of a report? Enumerate chronologically and
explain each of them in brief. 5
Answer:
a)

To:

Subject: An application for the post of IT officer

Sir,

I saw your online announcement for the post of IT officer, and I would like to apply for
the same. I have gone through your job descriptions, requirements and conditions. I have
come to realize that my academic profile and professional experiences will most
essentially match with your job requirements.

I have attached my CV along with this application. I am looking forward to your quick
response.
Regards
…………….

b) Supplementary parts follow the text of the report and provide information for readers
who seek more detailed discussion. Supplements are more common in long reports than
in short ones, and they typically include appendixes, a bibliography, and an index.
Appendixes: An appendix contains materials related to the report but not included in the
text because they are too long or perhaps not relevant to everyone in the audience. The
content of report appendixes varies widely, including any sample questionnaires and
cover letters, sample forms, computer printouts, statistical formulas, financial statements
and spreadsheets, copies of important documents, and multipage illustrations that would
break up the flow of text. An appendix is usually identified with a letter and a short,
descriptive title. All appendixes should be mentioned at appropriate places in the text
and listed in the table of contents.
Bibliography: To fulfil your ethical and legal obligation to credit other people for their
work and to assist readers who want to research your topic further, include a
bibliography, a list of the secondary sources you consulted when preparing your report.
This can also be called ―References‖ if it includes works consulted but not mentioned in
the report. In addition to providing a bibliography, some authors prefer to cite references
in the report text. Acknowledging your sources in the body of your report demonstrates
that you have thoroughly researched your topic. Furthermore, mentioning the names of
well-known or important authorities on the subject helps build credibility for your
message. Such source references should be handled as smoothly as possible.
Index:
An index is an alphabetical list of names and subjects mentioned in a report, along with
the pages on which they occur. If the readers need to access specific points of
information in a lengthy report, consider including an index that lists all key topics,
product names, markets, or important persons— whatever is relevant to the subject
matter. As with the table of contents, accuracy in an index is critical.

4. Write short notes on: (5×2=10)


a) Ethical communication
b) Stages of job interview
c) Barriers to effective listening
d) Web Conferencing
e) A follow-up letter

Answer:
a) Ethical communication
Ethics are the accepted principles of conduct that govern behavior within a society.
Ethical behaviour is a companywide concern, but because communication efforts are
the public face of a company, they are subjected to particularly rigorous scrutiny from
regulators, legislators, investors, consumer groups, environmental groups, labor
organizations, and anyone else affected by business activities. Ethical communication
includes all relevant information, is true in every sense, and is not deceptive in any
way. In contrast, unethical communication can distort the truth or manipulate
audiences in a variety of ways.

b) Stages of job interview


Job interview, an important component of employment communication, displays
distinctive features from other types of interviews. It is usually carried out in three
different stages: warming-up stage, question-answer stage and summing-up stage.
The warming-up stage refers to the stage that exists before the main conversation
between the interviewer and interviewee. In this stage both interviewer and
interviewee attempt to be prepared to lead the interview to a successful
communication. The interviewee is prepared with cheerful appearance and positive
thoughts and expectations. The interviewer attempts to make some kind of attachment
with the interviewee with the help of gestures, welcome note, etc. Similarly, in the
question-answer stage, the content based interaction between the interviewee and the
interviewer takes place. This is the largest stage of job interview. The final stage is
known as summing-up stage in which the interviewer signals pre-closing of the
conversation. And finally, the interview gets wrapped up with conventional thank-you
note and farewell exponents.

c) Barriers to Effective Listening


Barriers and noise can interfere with the communication process. There are various
barriers to effective listening and some of the prominent barriers are:
i) Physical barriers: We cannot listen if we cannot hear what is being said. Physical
barriers include hearing disabilities, poor acoustics, and noisy surroundings.
ii) Psychological barriers: Everyone brings to the communication process a different
set of cultural, ethical, and personal values. Each of us has an idea of what is right
and what is important. If other ideas run counter to our preconceived thoughts, we
tend to ―tune out‖ the speaker and thus fail to hear.
iii) Language problems: Unfamiliar words can destroy the communication process
because they lack meaning for the receiver. Similarly emotionally charged words
also cause miscommunication.
iv) Nonverbal distractions: Many of us find it hard to listen if a speaker is different
from what we view as normal. Unusual clothing, speech mannerisms, body
twitches, or a radical hairstyle can cause enough distraction to prevent us from
hearing what the speaker has to say.
v) Thought speed: Because we can process thoughts more than three times faster
than speakers can say them, we can become bored and allow our minds to wander.

d) Web Conferencing
Participants can take part in ―real life‖ meetings from the comfort of their offices.
Web conferencing is similar to videoconferencing but usually without the
transmission of pictures of the participants. They use their computers to access an
online virtual meeting room where they can present PowerPoint slides or share
spreadsheets or Word documents, just as they might do in a face-to-face meeting.
They can even demonstrate products and make changes in real time during a
meeting. Software such as WebEx and Microsoft Live Meeting makes Web
conferencing easy and effective.
e) A follow-up letter
In several situations, the follow-up letter is considered to be an important mode of
communication. It can be written after a business meeting, job interview, business
contract etc. In order to continue communicating with the recipient, you need to
write this type of a letter.

In the early stage of a business relationship, this type of a letter is written. It is


important to adopt the formal letter writing style. It is important to proofread your
letter in order to avoid spelling and grammatical errors.

Here are the two important reasons of sending a follow-up letter:


First is, it can offer an opportunity to maintain an additional contact after a
networking meeting, career, etc.

Second is, it can allow you to reconnect to the person with whom you are not in
touch for a while for the job search.
The follow-up letters are different from the thank you letters. The main purpose of
the letter is to show your thankfulness to someone for meeting you, approving
your project etc. It gives you the chance to re-introduce yourself.
Marketing
Maximum Marks –100
Total No. of Questions - 8 Total No. of Printed Pages -2
Time Allowed –3Hours
Marks
All questions are compulsory.
Section -'B'
5. Read the following case carefully and answer the questions given below: (45=20)
Shrestha tailoring center is a reputed brand in Kathmandu Valley for shirting
suiting and accessories. Thirty-two years ago, Shrestha tailoring center was
established by Babu Kaji Shrestha in Bagbazar. It is mainly famous for suits,
available both in readymade and through custom tailoring. It also offers casuals
and other wears for ladies and kids. ―We cater to the clothing of newborns to old
ones,‖ claims Shrestha. Apart from its main showroom in Bagbazar, it has
branches in several urban locations of Kathmandu such as Chabahil, and
Tripureshor.
The company‘s clients are mainly middle-class people. Though they have many
old clients, they also have increasing new customers. Apart from word of mouth,
it is also using advertisements from electronic media. Frequently, it offers free tie
for those who orders suit to attract customers. ―Last year, we offered free
Bangkok air tickets to lucky customers. We also give discount to customers
buying through debit and credit cards,‖ Shrestha said. It has also sponsored
several fashion shows, beauty pageants and programs in FM stations.
Price charged by the Shrestha tailoring center is slightly higher than other
competitors because of its location, quality clothing and brand image. To meet the
competition, they had offered suit at Rs. 4999 as a Dashain offer to attract price
sensitive customer. But this scheme did not work well. It became unsuccessful to
persuade economy minded customers. From the lesson learnt form the
unsuccessful Dashain scheme they are planning to modify the existing promotion
strategy. They are also planning to go beyond the Kathmandu Valley through
franchise and online business.
a) What is the main issue of the case?
b) Analyze the product strategy of Shrestha tailoring center.
c) Based on above case, identify the marketing concept implemented by Shrestha
tailoring center.
d) Are four P‘s sufficient for Shrestha tailoring center? Give your opinion.
Answer:
a) As per the above case, the main issue is implementation of inappropriate psychological
pricing as a promotional scheme. In the above case, reputed Shrestha tailoring center has
offered odd price i.e. Rs. 4999 to attract economy minded customers. Odd pricing targeted to
price sensitive customer is not matched with the brand personality of the tailoring center.
Thus, it failed to attract economy minded customers. It seems that tailoring center became
unsuccessful to attract new customer and to retain old customer because of inappropriate
pricing which is not in line with company image. Similarly, quality trap may be the reason as
customers perceive high price high quality and low price low quality. This type of appeal also
creates confusion among the quality conscious existing customer.
b) Shrestha tailoring center offers suits and accessories. It offers readymade suits and custom-
made suits. It also offers causals and other wears for ladies and kids. All products offered by
Shrestha tailoring center are closely related. They are closely related in terms of ultimate
consumption, in terms of distribution, in terms of promotion, in terms of target market. If any
company offers more than one product items which are closely related, that is popularly
known as product line strategy. Thus, based on product variety of company, we can say that it
is product line strategy.

c) In the given case, the company has used advertisement through electronic media. It has also
offered free goods i.e. tie for purchasing suits. It seems clear that they have also used sales
promotion tools like lucky draw i.e. free Bangkok air tickets. Several fashion shows, beauty
pageants and programs in FM stations have also been sponsored by Shrestha tailoring center.
It means they are using aggressive selling techniques to increase sales. The case is silent
about low price and easy availability or product quality. Similarly, the case is silent about
customer satisfaction and social responsibility.

Thus, we can conclude that based on given case, they are implementing selling concept of
marketing. As selling concept of marketing holds that customers will not buy their product
unless and until they are well informed. And aggressing selling techniques should be used to
increase sales and earn profit. It ignores customer satisfaction and social responsibility.

d) Marketing mix is popularly known as four P‘s which includes product, place, price and
promotion. Four P‘s are sufficient for physical goods but not sufficient for service products.
In today‘s context, it is very difficult to get pure physical goods or pure services. Features of
service are intangible, inseparable, perishable, indivisible and variable. Tailoring is a service
business. It offers hybrid product which includes both physical goods as well as service.
Thus, for customer satisfaction and organization goal achievement, Shrestha tailoring center
must not rely on four P‘s. They must address additional three P‘s i.e. people, process and
physical evidence while designing and implementing their marketing mix.

6. What is Promotion? What are the components of promotion? Explain the


objectives of promotion. (4+2+4=10)
Answer:
Promotion is the one of the major element of marketing mix which provides various valuable
information to consumers about product, its price, availability, utilities and benefits. Promotion
consists of various activities that facilitates exchanges with target customers through persuasive
communication which stimulates the demand of the product. Promotion is also known as
marketing communication.
According to Philip Kotler, ―Promotion includes all the activities the company undertakes to
communicate and promote its products to the target market.‖
According to Dictionary of Marketing, ―Promotion is the element in an organization‘s marketing
mix that serves to inform, persuade, and remind the market of a product and/ or organization
selling in the hope of influencing the recipients‘ feelings, beliefs, or behavior.‖
Promotion includes various types of marketing communication related activities to inform,
persuade, remind and reinforce the target market about the company‘s offerings which influence
the customer‘s feelings, belief or behavior of the customers.
Components of Promotion
There are mainly four components of promotion which are as follows:
• Advertising
• Personal selling
• Public relation and publicity
• Sales Promotion

Objectives of Promotion
The major objectives of promotion are informing, persuading, reminding and reinforcing or
reassuring the customer about the product.
1) Informing:
The main objective of promotion is to inform the market about product, price, availability,
utilities and benefits. It helps to develop awareness about the product It also provides
alternatives to the customers for purchase decision.
2) Persuading:
Promotion persuades customers to make the purchase decision in favor of the promoted
brand. Promotion is persuasive communication. It stimulates product demand through
appealing ads, incentives and benefits. Promotion influence buyer behavior. Promotion also
persuades middlemen to carry such product.
3) Reminding:
Customer normally has very short-lived memory. It is impossible to customers to remember
all advertised brand. So, marketer reminds customers about the product by using various
promotional tools. The marketer assumes that customer may forget unless they are constantly
reminded.
4) Reinforcing:
Promotion is equally important in post purchase stage of the buying process. Consumer may
feel anxiety after their purchase decision. Marketer often provides reinforcing message to
increase customer‘s satisfaction level. Repeated reinforcement also leads to brand loyalty. By
the reassurance to customer, marketer tries to reduce cognitive dissonance and build brand
and corporate image.

7.
a) What is distribution? Also point out the components of physical distribution. (3+2=5)
b) What is a package and packaging? Explain briefly the essentials of a good
package. (2+3=5)
Answer:
a) Distribution includes all the various activities the company undertakes to make the product
accessible and available to target customers. Distribution or Place is one of the important
elements of marketing mix. It includes the marketing channel and physical distribution.
Marketing channel makes the product available to the customers. Physical distribution makes
the product accessible to the channel members and customers. Distribution is also called the
―the other half of marketing.‖ It fulfills the gap between the producer and consumer.
Distribution channel is a set of interdependent organizations involved in the process of making
a product or service available for use or consumption by the consumer or business user.
Distribution channel may be direct channel, indirect channel or mixed channel. Physical
distribution is related with physical movements of goods from point of production to point of
consumption. It is also called market logistics.
Components of physical distribution are mentioned as follows:
i. Transportation
ii. Warehousing
iii. Inventory management
iv. Order processing
v. Material handling

b) Generally, packaging is an act of designing or producing the package for a product. In other
words, packaging can be defined as the process of designing the container for a product.
Packaging includes all activities required for designing and producing the container or
wrapper for a product.
While, a package means wrapper or cover or container in which a product is enclosed. We
see that some products are enclosed in a paper box some are enclosed in a tube, some are
enclosed into the canes, some are enclosed into a plastic bag, some are enclosed into a wood
box, and some are enclosed into a metal container, and so forth. All these are the types of
package. Use of a particular package largely depends upon the nature of product, demand of
the buyer and the particular country's legal requirements.

Essentials of a Good Package


A good package should pose the following characteristics:
i. It should be economical i.e., it should involve least cost to manufacture, to fill, and to
remove or to handle, so that it does not adequately affect the sales price of the product.
ii. It should be functional i.e., it should perform all those functions what it is supposed to do.
For example, is a package is used to minimize the cost of goods, it should do so. If a
package is used to attract the potential customers, it should do so. If a package is used to
protect from physical problems such as air, water, dust, etc., it should do so.
iii. In should be communicative or it should have promotional value i.e., it should
disseminate all information to the customers and other parties concerned regarding brand,
use and features of products, place and date of manufacture, name of manufacturer, etc.
iv. It should be attractive i.e., a package should be quite attractive since one of the main
objectives of packaging is to attract customers. In order to make a package attractive,
attractive and match-able color combination should be used, it should have good design, it
should use, as far as possible, attractive pictures or symbols or graphic presentation. A
right combination of these attributes will make a package more attractive, which will have
positive impact on company's sales volume and profitability.
v. It should be eco-friendly i.e., it should be environmentally friendly. A package should be
produced by such materials which do not affect environment or which do not create
environmental pollution. The green revolution has suggested the green marketers to use
environmentally friendly package. The actors of green revolution have suggested using
such package in the product, which not only must not contaminate the environment but
should protect it and even liquidate existing environmental damages. Many countries
including EU member countries, USA, Australia, and many more countries have their own
green legislations, which have implemented such conditions strictly. They can ban
importing those products from any country of the world, which do not meet their green
requirements.
vi. It should be durable i.e., a package should lengthen the life of the product so than the
marketers can store goods for a long period and consumers can use the product after a long
time from the date of manufacture.
vii. It should have commercial value i.e.; a package should be able to create adequate demand
for goods to maximize the sales quantity and earn adequate income.
8. Briefly explain the following: (5×2=10)
a) Market Segmentation
b) Pricing
c) Marketing Environment
d) Shopping Product
e) Meaning of E-marketing

Answer:
a) Market Segmentation
The process of defining and subdividing a large homogenous market into clearly
identifiable segments having similar needs, wants, or demand characteristics. Its objective
is to design a marketing mix that precisely matches the expectations of customers in the
targeted segment.
b) Pricing
Pricing refers to an act of determining the exchange value between what buyers get
and what the sellers receives. It is a process of setting prices for various products and
services. In short, pricing is the act of determining exchange value of a product or
service.

c) Marketing Environment
Marketing environment comprises all the actors and forces outside marketing that affect
marketing management‘s ability to develop and maintain successful transactions with its
target customers. Marketing is environment specific and change in marketing
environment creates opportunities and challenges in marketing. E.g. economic
environment, social environment, technological environment etc.

d) Shopping Product
Shopping products are less frequently purchased consumer products and services that
customers compare carefully on suitability, quality, price, and style. When buying
shopping products and services, consumers spend much time and effort in gathering
information and making comparisons. Shopping products marketers usually distribute
their products through fewer outlets but provide deeper sales support to help customers in
their comparison efforts. Clothing, furniture, motorbike are the examples of shopping
product.

e) Meaning of E-marketing
E-marketing or electronic marketing refers to the application of marketing principles and
the techniques through electronic media or internet. It is the process of marketing a brand
using the electronic devices that can be used to connect marketers and their customers via
the worldwide wave with the aim of attracting new customers retaining current business
and developing its brand identity. It is useful to exploring domestic as well as
international market through internet.
Income tax & Vat
Maximum Marks - 100
Total No. of Questions - 8 Total No. of Printed Pages -6
Time Allowed - 3 Hours
Marks
Attempt all questions. Working note should form part of the answer.
1. I & M Pvt. Ltd. Kathmandu, a manufacturing company, deals in production and sales of
garments. Based upon the following information, you are requested to ascertain taxable
income and tax liability for Income Year 2073/74. 20

Particulars Amount (Rs.)


Export Sales 3,00,00,000
Misc. Income 25,00,000
Dividend Income 10,00,000
Opening stock 75,00,000
Raw material import 70,00,000
Freight for raw material 9,00,000
Custom duty paid for raw material 15,00,000
Custom agent fee 1,50,000
VAT paid for import 15,00,000
Wages for production 30,00,000
Overhead cost 20,00,000
Administrative expenses 15,00,000
Selling and distribution expenses 10,00,000
Interest paid to Financial Institution 3,00,000
Penalties paid to Metropolitan Office 2,00,000
Donation 2,00,000
Depreciation 12,00,000

Additional information:

a) Sales include VAT refund of Rs. 10,00,000 paid at custom point for import of raw
material. Miscellaneous income Rs. 20,00,000 is the incentive given by the Nepal
government for export of previous year and is received during this year through Nepal
Rastra Bank. Remaining Miscellaneous income Rs. 5,00,000 has been generated from
the bank deposit.

b) Dividend income has been received from Joint Venture.

c) The opening stock and closing stock has been recorded 1,000 pieces and 1,200 pieces
respectively. The opening stock includes factory fixed overhead Rs. 10 per piece and
repair and maintenance Rs. 15 per piece. During the year, the company produced
2,000 pieces of garments and the overhead cost includes equal rate of previous years
fixed overhead and repair and maintenance cost.

d) Administrative expenses, includes travelling cost Rs. 1,00,000 of employees during


the import of raw material, entertainment expenses Rs. 50,000 provided to the
Custom's employee and custom agent during the import.

e) The company has the policy for employee bonus; annually it makes 10 % provision
from the profit as per the Bonus Act. During this year, it has paid Rs. 2,00,000 and
booked under the administrative expenses.
f) Selling and distribution expenses were paid to the cargo agent for export, invoices
amounting Rs. 2,00,000 from the cargo agent were received, and the remaining
amount pertaining to the transportation of the goods from the factory to the depot
point as per details given by the agent.

g) Interest expense has been charged against the loan taken to purchase the factory
machinery costing Rs. 20,00,000. The machinery was purchased on 1st Ashadh, 2074
and has been used from 29th Shrawan, 2074.

h) The donation includes Rs. 1,00,000 directly paid to the flood victims; the remaining
amount was deposited into Reconstruction Fund established by Nepal government for
earthquake victims.

i) Opening balance (WDV) of depreciable assets as on 2073.04.01 as per books of


account are as follows:
a. Land Rs. 20,00,000
b. Building Rs. 50,00,000
c. Cars Rs. 26,00,000
d. Plant & Machinery Rs. 80,00,000
The company disposed a Santro car having written down value of Rs. 10,00,000 for Rs.
8,00,000 during the year.

Answers:
Particulars Sec. Note Amount
Inclusions:
Sales 7 (2) 1 29,000,000
Miscellaneous Income 7 (2)
Export Incentive 2.1 -
Interest income directly related to 2.2 500,000
business
Dividend Income 7 (3) & 92 3 -
Total Inclusions (A) 29,500,000

Deductions
Interest Expense 14 4 -
Cost of Trading Stock 15 5 13,293,000
Depreciation 19 6 2,413,560
Repair & Improvement Cost 16 7 30,000
General Deductions
Administrative expenses 13 8 1,250,000
Selling & Distribution Expense 13 9 200,000
Provision for Employee Bonus 13 10 531,818
Penalties paid to Metropolitan Office 21 11 -
Total Deductions (B) 17,718,378
Assessable Income from Business 11,781,622
(A-B)

Calculation of Taxable Income


Assessable Income from Business (as 7 11,781,622
above)
Total Assessable Income 11,781,622
Less: Allowable Reductions
Donation to Exempt Entities 12 12 -
Contribution to National 12B 12 100,000
Reconstruction Fund of GON

Taxable Income 11,681,622

Calculation of Tax Liability


Taxable Income 5 11,681,622
Tax Rate 13 15%

Tax Liability 1,752,243


(Alternative Tax Computation)
Taxable Income from Export @15%
Taxable Income from Other @25%

Notes & Working Notes:


1. Calculation of Sales as per Tax
Particulars Amount (Rs.)
Export Sales (given) 30,000,000
Less: VAT refund included in Sales (Note 1.1) (1,000,000)
Sales u/s 7 (2) (b) 29,000,000

1.1. VAT is refundable duty and does not form part of either cost of sales when paid or
income when refunded.

2. Miscellaneous Income
2.1. Export Incentive:
As per Sec. 22 of Income Tax Act, 2058; company must follow accrual basis of accounting. As such, the
export incentive in relation to accrual of previous income year is part of income of previous income year
and not of this Income Year. Thus, the amount does not form part of business income of IY 2073/74 as
per Sec. 22.
(Alternatively, if students assume that the right to receive the export incentive was established during
current Income Year and hence, treated as accrued during the Income Year, the amount may be included
in current year's income with appropriate note regarding the same)

2.2. Interest income, though return of investment in nature, but is related to business objective of the company
and hence included in income as per Sec. 7 (2) (g) of the Income Tax Act.

3. The joint venture distributing dividend is assumed to be resident. As such, dividend income received from
resident joint company (Joint venture is a company as per definition) is a final withholding payment as
per Sec. 92 of the Act. Though it is investment income directly related to business activity of the person
and seems to be included u/s 7 (2) (g) but since it is final withholding, it must not be included in income
as per Sec. 7 (3) of the Act.
(Alternatively if student assume that the Joint venture is a nonresident joint venture, the amount may be
included in income)

4. As per Sec. 14 (1), if interest is accrued on loan borrowed for the purchase/creation of asset during the
income year, such interest expense shall be claimed only when the related asset is used during the income
year. Since, the asset is not used during the year; the interest expense is not deductible.
5. Calculation of Cost of Trading Stock
Particulars Amount
Opening Stock 7,500,000
Less: Adjustment for Repair & Maintenance Expense (15,000)
(15*1000)
Add: Cost of Production
Import of Raw Material 7,000,000
Freight for Raw Material 900,000
Custom duty paid for raw material 1,500,000
Custom agent fee 150,000
Wages 3,000,000
Overhead Expense (assumed to be production overhead) 2,000,000
Less: Adjustment for Repair & Maintenance Expense in cost of
(30,000)
production
(2000*15)
Less: Value of Closing Stock (8,712,000)
(Assumed FIFO)
(Cost of Production divide by 2000 multiplied by 1200)
Cost of Trading Stock 13,293,000

5.1. Refundable duties do not form part of cost of inventories. As such, VAT paid on import is not part of cost
of trading stock, and hence, not included.

6. Calculation of Depreciation Expense


Particulars Pool A Pool C Pool D
I. Depreciation Rate 6.67% 26.67% 20.00%
II. Opening Depreciation Base
2,600,000 8,000,000
5,000,000
III. Absorbed Additions
IV. Disposals 800,000
V. Depreciation Base
5,000,000 1,800,000 8,000,000
VI. Depreciation Expenses
333,500 480,060 1,600,000
VII. Total Depreciation Expense 2,413,560

7. Calculation of Eligible Repair & Improvement Cost


Particulars Pool A Pool C Pool D
I. Depreciation Base
5,000,000 1,800,000 8,000,000
II. 7%of Depreciation Base
350,000 1,26,000 560,000
III. Actual Repair & Improvement Cost
- 0 30,000
IV. Eligible (Lower of ―II‖ or ―III‖)
- - 30,000
Alternatively, full marks shall be granted for repair calculation only for
specific pool and not for all pools.

8. Eligible Administrative Expenses


Particulars Amount
Given 1,500,000
Less: Entertainment Expense Note 8.1 (50,000)
Less: Bonus Paid for previous year
erroneously charged as expense Note 8.2 (200,000)
1,250,000

8.1. Since the amount is paid to custom employee and custom agent without proper receipt of service and
proper justification for such payment, the amount is a cost not for business purpose, thus, is not a
deductible expense u/s 13 (c) and Sec. 21 (1) (f).

8.2. Company must follow accrual basis of accounting, as such, the bonus expense accrued for last year and
paid during the year included as part of administrative expense is not deductible expense u/s 13. (It is an
accounting error and must be rectified, which decrease administrative expense for accounting purpose as
well)

9. Eligible Selling & Distribution Expenses


Particulars Amount
Given 1,000,000
Less: Ineligible due to absence of
supporting (800,000)
Given 200,000

9.1. The expense without supporting is ineligible as the purpose of expense cannot be justified due to lack of
evidence justifying the purpose.

10. Calculation of Bonus Expenses


Accounting profit 58,50,000
Bonus Expense for the Year 531,818 Based on New Circular of IRD
(Profit divided by 1.1 &
multiplied by 0.1)

11. Penalty is not a deductible expense as it is paid as a result of infringement of law u/s 21 (1) (b).

12. Since direct payment to flood victim is not a payment of donation to exempt organization, thus the
amount is not deductible. The contribution to National Reconstruction Fund of GON is eligible for
deduction u/s 12B.

13. Calculation of Effective Tax Rate


Since it is a special industry, the applicable tax rate as per Schedule 1, Section 2 (3) is 20%. As it is
export oriented industry, it is entitled to exemption of 25% on tax rate as per Sec. 11 (3E) and as such, the
effective tax rate is 15%.

2. Mr. Tukaram Bidari joined Star General Stores, a trading firm on 1st Shrawan 2070. The
following information is received in relation to his employment income for the Income
Year 2073-74:
a) Pay scale Rs. 22,000 - 1,500 - 40,000.

b) He was paid with festival allowances equal to one months‘ salary. Apart from the
allowance, the firm provided 1 set TV as a gift to the best performing staff. The
market value of the TV is Rs. 55,000.
c) The firm pays Mr. Bidari Rs. 15,000 p.m. as house rent allowance. Out of the
allowance, he pays only Rs. 12,000 p.m. as house rent and saves Rs. 3,000 p.m.

d) He is entitled to entertainment allowance of Rs. 500 p.m. and annual medical


allowance equal to 2% of salary.

e) The firm paid salary of Rs. 6,000 to the cook of Mr. Bidari in his residence. He
compensated Rs. 2,000 to the firm on this account.

f) Appointed in a senior post, he attended 12 meetings and was paid Rs. 2,000 per
meeting as meeting allowance. He also received allowance Rs. 2,000 p.m. for writing
minutes of Human Resource Committee of the firm.

g) He was provided meal in the office in an equal term to all employees of the firm. The
expenses borne by the firm for this facility was Rs. 36,000 for entire year i.e. 3,000
p.m.

h) He spent Rs. 20,000 for promotion of firm by hosting dinner to customers. Cost on
this account was reimbursed by the firm to Mr. Bidari.

i) The firm paid Rs. 1,000 p.m. as tuition fees for each of two children of Mr. Bidari.
This payment was made directly to school rather than to Mr. Bidari.

j) During the year, he was paid Rs. 10,000 on account of leave encashment. Further, at
the end of the year, his accumulated leave pay was valued at Rs. 50,000 which was
payable on retirement from the firm.

k) Mr. Bidari received a sum of Rs. 30,000 from his previous employer. The payment
was made on account of target commission on sales achieved by him when he was in
employment with previous employee.

l) Mr. Bidari paid life insurance premium as follows:


 Rs. 15,000 for his policy of Rs. 5,00,000
 Rs. 10,000 for policy of Rs. 2,50,000 of his son

m) Mr. Bidari met an accident during the financial year and received Rs. 5,00,000 as
compensation from insurance company, out of which he incurred Rs. 1,00,000 for
medical treatment. Mr. Bidari had contributed only Rs. 1,50,000 as insurance
premium to insurance company so far.

You are required to calculate income from employment and net tax liability of Mr. Bidari
for income earned during FY 2073/74. 10

Answers:
Particulars Note Amount (Rs.)
Basic Salary
318,000
(22,000 + 1500 ×3 ) × 12
Festival Allowance 26,500
Gift 55,000
House Rent Allowance 180,000
Entertainment Allowance 6,000
Medical Allowance (2% of Salary) 6,360
Cook Facility 4000
Meeting Allowance 1 -
Allowance for minutes 24,000
Notes:
Meal Facility 2 - 1. M
Reimbursement of Business Promotion Exp 3 - eeting
Tuition Fees Paid 24,000 allowa
Provident Fund Contribution by employer nce is
Leave Encashment 4 10,000 final
Accrued Leave Provision 5 withho
Commission from previous employer 6 30,000 lding.
Compensation against Accident 7 0 2. A
Assessable Income from Employment 6,83,860 s per
Sec. 8
(3),
Assessable Income
Meals
Less:
or
a. Contribuion to approved retirement fund least of i, ii and
lunche
iii below:
s
i) Actual contribution (Rs. 63,600)
provid
ii) Rs. 300,000
ed to
iii) 1/3 of Assessable income (1/3 for Rs. 7,59,660)
all
Taxable Income 6,83,860
employ
Allowable Deduction under Schedule 1:
8 15,000 ees
Life Insurance Premium Lower of i & ii
under
i) Rs. 20,000
equal
ii) Actual Premium (Rs. 15,000)
terms
Balance Taxable Income 6,68,860
at
First ( 1 % Social Security tax up to first Rs. 3,50,000) 3,500 workpl
Next (15 % up to next Rs. 100,000) 15,000 ace are
Remaining Rs. 2,18,860 @ 25% for (up to next Rs. not
54,715
2,000,000)
include
35% above Rs. 2,500,000 -
d.
Total Tax Liability 73,125 3. R
eimbursement of expenses serving proper business purpose of employer does not form part of
employment income (as per Sec. 8 (3)).
4. Leave encashment paid during the tenure of employment forms part of employment income as it is
not a retirement payment.
5. Natural person must follow cash basis of accounting when calculating his/her income from
employment (as per Sec. 22), as such, accrued leave provision is not included in income.
6. Employment includes past, present and prospective employment, and computation of employment
income must be done under cash basis of accounting. As such, commission received in cash during
the year from past employer is included as part of employment income.
7. As per proviso clause (a) of Sec. 31, Compensation received against physical injury of a natural
person shall not be included in income.
Medical tax credit is not allowed on medical expenditure incurred for treatment of physical injury
using compensation received as per Sec. 31.
8. Payment of investment insurance premium for the insurance of his son is not deductible under Sec. 1
of Schedule 1.
3.
a) Oriental Pvt. Ltd is dealing with buying, constructing and selling of land and building.
It has the following transactions for one of the projects during the year 2073/74.

Land purchase for the project Rs. 50 million


Land purchase for office Rs. 10 million
Housing construction cost (10 houses) Rs. 150 million
Building construction cost for office Rs. 10 million
Land in Stock Rs. 20 million
Housing in Stock Rs. 60 million
Sales of housing (8 houses) Rs. 300 million

Based on the above information, answer the following questions: 5

i) Calculate gain and loss in this transaction,

ii) Whether the company should pay capital gain tax or not?

iii) If the company should pay capital gain tax, how much and where to pay?

iv) If the company does not require to pay the capital gain tax, then what and where
to pay tax on the gain of this transaction?

b) Mr. Ram has been holding shares of various listed companies of Nepal from primary
& secondary markets. Total cost of the shares is Rs. 2 million. He left for USA for a
year on Aswin 30, 2070 but he returned back on Kartik 15, 2072. The market value of
the shares was Rs. 3.5 million at the time he became non-resident.

He sold all the shares for Rs. 3.6 million on Kartik end, 2073. He has received his
payment from share broker after deducting broker commission Rs. 20,000. Advance
tax is deducted by NEPSE as per Section 95Ka (2). Compute gain and loss under
Section 40 (3), gain and loss calculation on Kartik end 2073 by NEPSE and tax
withholding amount in these situations. 5
Answers:
a)
a. Calculation of Gain and loss
The purchase of land and building for official purpose is not considered for calculation of gain
and loss, because land is business assets and building is depreciable asset. Trading stock is taken
into account to calculate the gain and loss:

Incomings:
Sales of housing (8) Rs. 300 Million
Less: Outgoings:
Cost of sales of housing Rs. 90 Million (Rs. 150 M-Rs.60 M closing stock)
Cost of land Rs. 30 Million (Rs. 50 M- Rs.20 M closing stock)
Gain Rs. 180 Million
b. As per section 95ka (6), capital gain tax @ 10 % on capital gain shall be applicable on the
disposal of land and building of any person other than natural person. So, the company should pay
the capital gain tax.
Alternatively, if the student may refer to IRD Circular (without referring it) and conclude that
since it is a trading stock, such advance tax collection is not required.
c. The company should pay the capital gain tax amounting of Rs. 18 Million (10 % of Rs. 180
Million) at the time of registration to the Land Revenue Office.
Alternatively, if student refer to the non applicability of advance tax collection on trading stock
(land and building of real estate company, the amount of capital gain is Zero.

d. Company is not relieved from taxation on its business income (i.e. income from real estate
dealing) even when the company is not required to pay advance tax at the time of registration of
land and building at Land Revenue Office. As such, the company is required to make calculation
of tax liability for the Income Year under Self Assessment System and pay such self assessed tax
along with submission of income return. The payment shall be made in installments and any
residual unpaid balance of tax liability not covered by installment tax shall be paid as and when
the return is filed.

b)

As per Section 40(3) of Income Tax Act, assets and liabilities shall be considered to have disposed
immediately before that person becomes a non-resident person, except for the land or building
situated in Nepal. As per the provision, gain and loss on the shares shall be calculated as follows:

Computation of gain/loss at the time of becoming nonresident

Incomings
Market Value at the time of becoming nonresident (Sec. 41 (1) (a) Rs. 3.5 million
Incomings Rs. 3.5 million

Amount paid at inception Rs. 2.0 million


Amount paid during holding period Rs. 0.00
Amount paid on disposal Rs. 0.00
Outgoings Rs. 2 million

Gain on deemed disposal Rs. 1.5 million

Tax rate applicable 5%


TDS amount 75,000

As the Nepalese Tax system follows Self Assessment process, a natural person becoming nonresident
shall file jeopardy self assessment income return as per Sec. 100 (1) at the time of becoming
nonresident and clear all applicable taxes until such period.

When he sold all the shares, the person shall disclose his Outgoings for the shares as Rs. 3.5 Million
to NEPSE and require NEPSE to collect advance tax as follows as per Sec. 95A:

Incomings from Shares Rs. 3,600,000


Less: Outgoings
U/s 41 (1) (b) Rs. 3,500,000
Broker Commission at the time of actual disposal Rs. 20,000
Gain (Incomings – Outgoings) Rs. 80,000
Tax rate applicable 5 percent
Advance tax collected by NEPSE from shareholder Rs. 4,000
(Advance Tax Collection Rate is 5% in both the case since the person is resident at the time of deemed
disposal and actual disposal)
4.
a) Comfort Airways Pvt. Ltd. has been established in FY 2068/69 with total paid up
capital of Rs. 1.5 billion. Currently, as the domestic airliner, it has been operating all
flights by 4 Beach Crafts to different tourist destinations. These Beach Crafts are
acquired directly from the manufacturer on Operating Lease.
Of late, the management of the company is very much certain to earn additional profit
Rs. 7.5 million at the end of FY 2073/74. To achieve this target, the company plans to
acquire one more 77 seater ATR air craft on finance lease for 15 years period to
expand its wings to India. This is possible subsequent to the intake of additional paid
up capital of Rs. 75 million for which all the shareholders are ready to invest.

As a tax consultant, the management wants to know any tax implication to the
company by adding the aircraft. 5

b) Mention whether the return filing is required or not on the following circumstances: 5

i) Mr. X, a natural person having employment income of Rs. 4,00,000 in 2073/74,


has worked under a foreign consultancy firm, i.e. non-resident for Nepalese tax
purpose. Assume no deduction of retirement fund, insurance, remote area
allowances etc.

ii) Mr. X with Mrs. X, a resident natural person having employment income of Rs.
2.1 million (Rs.1 million of Mr. X and Rs. 1.1 million of Mrs. X) in the year
2073/74. Assume no deduction of retirement fund, insurance, remote area
allowances etc.

iii) S Enterprises, Kathmandu registered in Small and Cottage Industries Office is


dealing with VAT exempt goods. It has annual sales Rs. 15,00,000 in 2072/73 and
Rs. 25,00,000 in 2073/74. It has assessable incomes Rs. 1,50,000 and Rs. 1,00,000
in 2072/73 and 2073/74 respectively.

iv) V Enterprises Kathmandu registered in Small and Cottage Industries Office is


dealing with VAT exempted and taxable goods. It has annual sales Rs. 5,00,000 in
2072/73 and Rs. 30,00,000 in 2073/74. It has assessable income Rs. 1,00,000 and
Rs. 5,00,000 in 2072/73 and 2073/74 respectively.

v) Mrs. Chaulagai, a single natural person has income from various sources like
dividend income Rs. 2,00,000, interest income Rs. 3,00,000 from bank, meeting
allowances Rs. 20,000 and capital gain from sale of shares Rs. 3,00,000.

c) Quantify the value of perquisite as per the provision of Income Tax Act, 2058
provided by PQR Ltd. to the employee and others as under: 5

i) The company pays Rs. 35,000/month for a furnished accommodation facility to


the Executive Officer (Mr. Lama). The value of the furnitures and equipments is
Rs. 1,50,000. Mr. Lama‘s basic salary and grade was Rs. 1,00,000 p.m. during the
financial year 2073/74.

ii) The company pays Rs. 15,000/month for a flat provided to Program Officer, Mr.
Tamang. Mr. Tamang‘s basic salary and grade was Rs. 50,000 per month during
the financial year 2073/74.
iii) The company pays Rs. 1,00,000/month for a flat provided to Mr. Thapa, a board
director. The value of the furniture and equipment provided by the company in the
rented flat was Rs. 5,00,000.

iv) Free four wheeler facility is provided to the Executive Officer (Mr. Lama). The
company meets the cost of fuel and maintenance charge of the vehicle provided to
Mr. Lama. The cost of fuel and maintenance charges paid by the company was
Rs. 2,50,000 and the value of four wheeler was Rs. 75,00,000. Mr. Lama‘s basic
salary and grade was Rs. 100,000 per month during the financial year 2073/74.

v) The company meets the cost of fuel and maintenance charge of vehicle provided
to Mr. Thapa, the board director. The cost of fuel and maintenance charges paid
by the company was Rs. 3,00,000 and the value of four wheeler was Rs.
1,00,00,000.

d) Oppo International Pvt. Ltd., Kathmandu is a dealer of Oppo Electronics. The Oppo
Electronics are manufactured in China by Oppo International. The company imported
the electronics goods worth of US $ 50,000 on 23 Baishakh, 2074, the payment was
done on 23 Shrawan, 2074. The exchange rate of the transaction was as follows:

Baishakh 23: Rs. 104 per US $


Closing of Ashadh: Rs. 107 per US $
Shrawan 23: Rs. 106 per US $

You are required to compute the amount for inclusions or deductions for the tax
assessment of this transaction mentioning the relevant provisions of Income Tax Act,
2058. 5
Answers:
a)

As per the provisions of Section 11 (3 ta), the Airline company operating international flights with an
investment more than Rs. 2 billion are inter alia privileged with 100% tax holiday for the period of 5
years from the date of operation, and thereafter privileged with 50% rebate on tax for the period of 3
years.

However, the privileges as aforesaid are also available to the existing Airline Operators provided they
enhance their existing installed capacity by at least by 25%, and increase their investment to at
least Rs. 2 billion. Such privileges commences immediately on the fulfillment of the two conditions
as aforesaid. In such case, the tax holiday and rebate shall be provided to such income that is
generated from enhanced capacity.

In the case of our problem, Comfort Airways Pvt. Ltd. with the increased installed capacity of
operations by two more international sectors, and total investment of Rs 1.5075 billion will not have
privilege to enjoy tax holidays as well as tax rebate as aforesaid.
b)

i) A person is not required to file return in any of the following conditions unless required by IRD in
writing or through public circular or the total income (including taxable or not except interest of
final withholding in nature and meeting allowance) exceeds 40 Lakhs:
 Under Section 97 along with Section 4(3), resident natural person having salary income from
resident employer with no claim of donation, more than medical tax credit and retirement
contribution, need not to file income tax return, or
 a person having no income tax liability, or
 a natural person having taxable income from operation of vehicle as specified in Sec. 1 (13) of
Schedule 1, or
 person deriving exclusive income from final withholding payment

In this case, since any of the above condition is not satisfied, Mr. X has to file income tax return of
his salary income for the income year.

ii) A person is not required to file income return as explained in answer (i).
Since both the spouses meet the conditions as prescribed by Sec. 4 (3) of the Income Tax Act,
2058 and individual income does not exceed Rs. 40 Lakhs, each of the spouses is not required to
file income return (as it is obvious that a person does not elect to file couple when both the spouses
are earning member since we need to presume that each taxpayer is rational and uses benefits
provided by tax law)
In case a student presumes application of Sec. 50 by spouses and filing jointly (which is absurd in
case of a rational taxpayer), any of the conditions in Sec. 97 (1) will not be satisfied, requiring
them to file income return.

iii) Since, there is tax liability in case of S Enterprises for both the Income Years (whether electing
Sec. 4 (4) or making payment of taxes u/s 4 (4Ka)), and any of the other three conditions as
specified in Sec. 97 (1) (part (i) of this answer) is not satisfied, S. Enterprises require to file tax
return.

iv) Since, there is tax liability in case of V Enterprises for both the Income Years (whether electing
Sec. 4 (4) or making payment of taxes u/s 4 (4Ka)), and any of the other three conditions as
specified in Sec. 97 (1) (part (i) of this answer) is not satisfied, V Enterprises require to file tax
return.

v) Section 97 (1) (kha) states that if a person is related to the section 3(c) the person is not required to
file the income tax return for the income year. Section 3(c) is the provision for the final
withholding payments. As per these provisions, Mrs. Chaulagai is not required to file for the
income of dividend, interest, meeting allowances.

But since she has capital gain income, she is required to file income return.

c) Under Section 27: Quantification of the value of perquisite provided by PQR Ltd. to employee and
others as per the provision of IT Act.
i) Rs. 24,000
2% of salary in case of salaried person i.e. (2% of {12 x (Rs. 1,00,000)}
ii) Rs. 12,000
2% of salary in case of salaried person i.e. (2% of {12 x (Rs50,000)}
iii) Rs. 300,000
25% of rent paid or imputed rent for own house for other person i.e. (25% of (12 x Rs. 100,000)
iv) Rs. 6,000
0.5% of salary in case of salaried person i.e. (0.5% of {12 x (Rs. 1,00,000)}
v) Rs. 100,000
1% p.a. of market value of vehicle per annum i.e. (1% of x Rs. 1,00,00,000)

d)

Conversion to Rupees under section 28, Exchange differences under section 24 (4) and claim for cost
of sales under Section 15 are related for these transactions.

The company should maintain its books of accounting in accrual basis under section 22 of the Act.
Further, the section 24 (4) has mentioned the accounting for exchange difference. The provision is as
follows:

In calculating a person's income earned from a business or investment on the accrual basis, where the
person includes any payment to which the person is entitled or deducts any payment that the person is
obliged to make and a difference in the payment received or made by the person occurs as a result,
inter alia, of a change in exchange rate, the amount of difference shall be adjusted at the time when
payment is received or made.

So, the company should account in Nepalese currency and calculate foreign exchange gain or loss.

Computation of inclusions or deductions


Based on the above provision, the company can include purchase amount of Rs. 52, 00,000 (US $
50,000 *104) in income year 2073/74 while calculating the cost of sales under 15. The exchange loss
of Rs. 100,000 shall be claimed deduction under 13 in income year 2074/75.
(Alternatively since exchange loss is related to loss on disposal of liability, the students may also refer
the same in answer.)

5. Write short notes (4×2.5=10)


a) Valuation of Closing Stock
b) Associated Person
c) Tax Payer‘s Rights
d) Self-assessment

Answers:
a) Valuation of closing stock of a business for income year is done at a lower of the following:
i) Cost of closing stock that remains at the end of year or
ii) Market price of the closing stock at the end of year;

Where identification of each items stock is not possible Income Tax Act permist first in first out
(FIFO)
OR weighted average method for valuation of closing stock

The cost of closing stock (trading stock) shall not include repair and improvement cost and
depreciation of depreciable asset.

b) As per section (ka na) of the Income Tax Act, 2058, Associated person means
a. a person or
b. more than one person or
c. a group of such persons who acts as per intention of each other,
And includes –
a. Natural person and relative of such natural person, or any person and partner of such person
b. Foreign Permanent Establishment and a person having ownership in such establishment
c. Any entity either directly or indirectly through
(a) other person or
(b) related entity or
(c) other person or entities related with such related entity,
Have control over or are entitled to have benefit of 50% or more than 50% of income, capital or
voting power of any entity.
But below persons shall not be treated as associated person
a. Employee,
b. Any other person as specified by the Department
c) As per the Section 74(2) of Income Tax Act 2058,following rights are available to the tax payers:

i) Right to get respectful behavior


ii) Right to receive any information related to tax as per the prevailing laws
iii) Right to get an opportunity of submitting a proof in one‘s own favor as regards tax matter
iv) Right to appoint lawyers or auditors for defense, and
v) Rights to secrecy in respect of tax matters and to keep it inviolable.

d) As per the provision (Sec.96) of Income Tax Act 2058, the tax payer has an obligation to submit the
tax return on time.
Actual Self Assessment:
Under (Sec.99), the person submits a tax return including the information regarding the total tax
payable during the year, and the tax due for payment on the date of submitting the return; it is believe
that the income tax assessment is complete
In general terms, the filling of an annual return is the self-assessment made by the tax payer that is
treated as assessment unless the Jeopardy Assessment (u/s100) & Amended Assessment (u/s 101)
prevail.
Deemed Self Assessment:
Even in the case of a person who fails to file the annual tax return, the income tax for the year is
deemed as assessed on due date of Asoj-end under the following basis:

i) The total tax liability of the tax payer during the year is equal to the amount of withholding tax
deducted by withholding agents on its payments, and the amount of advance tax paid by it.
The deemed tax assessment shows that there is no more tax payable for the year by such tax payer.

6. Namaste Steel Private Limited, a VAT registered Company, has witnessed stiff downfall
in demand of its products, and hence its operation has been shut down since the past 3
months. The company is considering for closure of the business.

Below is the detail of its assets and liabilities, and VAT receivable payable:
a. As per VAT return filed for the previous VAT period, it has VAT credit of Rs. 1
million.
b. Below is the detail of its stock and capital goods and other assets and liabilities as on
the immediately previous VAT period:

i. Finished Goods Inventory (VAT applicable) – Rs. 5 million (Market Value Rs. 4
million)
ii. Raw Material (VAT applicable) – Rs. 1 million (Market Value Rs. 1.5 million)
iii. Damaged Goods (VAT applicable) – Rs. 5,00,000 (Market Value Rs.
1,00,000)
iv. Plant and Machinery (VAT applicable) – Rs. 9 million (Market Value Rs. 4
million)
v. Vehicle – 4 Wheeler (VAT applicable) – Rs. 3 million (Market Value Rs. 4
million)
vi. Vehicle – 2 Wheeler (VAT applicable) – Rs. 1,00,000 (Market Value Rs.
1,00,000)
vii. Land – Rs. 20 million (Market Value Rs. 50 million)
viii. Sundry Debtors – Rs. 6 million (Rs. 3 million is bad debts)
ix. Sundry Creditors – Rs. 4 million (all amount payable)

Suggest the management of the company regarding: 10


a) The provisions in the Value Added Tax Act for cancellation of registration of VAT,
and
b) Amount of VAT payable or refundable to the company.

Answers:

As per Sec. 11 (1), the conditions for Cancellation of Registration are:


a. In case of an organized institution: In case of closure, sale or transfer of the organized institution or the
cessation of the existence of the institution due to any reason
b. In case of ownership of Individual: In the event of death of such individual
c. In case of a partnership firm: In case of dissolution of partnership or the death of a partner
d. In case a registered person leaves conducting taxable transaction
e. In case of a taxpayer who submits Zero Return continuously for a period of one year or does not submit
VAT Return at all
f. In case where the total taxable turnover of a person does not exceed Rs. 5 Million (2 million in case of
service business or mixed business of goods and services) in preceding consecutive 12 months period
g. In case of registration in error

As such, as per Section 11 (1) (a) as the company is considering closure, the VAT registration may be
cancelled when a closure notice is filed with concerned tax officer. For Namaste Steel Private Ltd. to go
for immediate cancellation of registration, it may do so if its turnover of preceding consecutive 12 months
is Rs. 50 Lakhs or less. Unless any of these two conditions is satisfied, it cannot make an application for
cancellation of registration.

For this, following procedure must be applied:


a. VAT registered entity should file application in a prescribed form stating reason for cancellation of
VAT registration for cancellation of Vat registration within 3 months of the date of occurrence of
events mentioned in section 11(1) of Value Added Tax Act, 2058. (Rule 12(1) of Value Added Tax
Rule, 2053)
b. Such entity should file VAT return till date along with VAT amount payable if any. (Rule 12(1) of
Value Added Tax Rule, 2053)
c. Such entity should pay VAT on closing stock of goods (including capital goods) in which VAT credit
had been taken earlier presuming the same has been disposed off at the market value. (section 11(4) of
Value Added Tax Act, 2052)
d. Tax officer shall carry out tax assessment, and decide for whether the VAT registration should be
cancelled or not within 3 months of date of such application. (section 11(3) of Value Added Tax Act,
2052)
e. However, VAT registered person shall file VAT return for next 3 months or date of decision of VAT
cancellation whichever is earlier. (Rule 12(3) of Value Added Tax Rule, 2053)
f. However, such person cannot be relieved of any liability which may be incurred due to something
which has been done which should not have been done or something which has not been done which
should have been done during the period when such person was registered under VAT simply due to
the reason that VAT registration of such person has been cancelled.

Hence, following the above provisions of the law, Namaste Steel Private Limited can cancel its‘ VAT
registration.

Computation of VAT Payable/Refundable to the Company


Particulars Market Value VAT @13%
VAT on closing stock assuming credit already taken
Finished Goods Inventory 4 million 0.52 million
Raw Material 1.5 million 0.195 million
Damaged Goods 0.1 million 0.013 million
Plant and Machinery 4 million 0.52 million
Vehicle – 4 Wheeler 40 Lakh 0.52 million
Vehicle – 2 Wheeler 0.1 million 0.013 million
Land VAT not applicable 0
Sundry Debtors VAT not applicable 0
Sundry Creditors VAT not applicable 0
Total VAT Payable 1.781 million
Opening VAT Receivable 1.0 million
Net VAT Payable 0.781 million

7.
a) Silver Imports and Exports, Kathmandu has the following transactions of garments
and ginger:

Particulars Jestha, 2074 Ashadh, 2074 Shrawan 2074 (Rs..)


(Rs.) (Rs.)
Garments Export 3,00,000 6,00,000 10,00,000
Garments Local Sales 7,00,000 6,00,000 5,00,000
Ginger Export 1,00,000 3,00,000 4,00,000
Ginger Local Sales 4,00,000 5,00,000 6,00,000
Garment Purchase 8,00,000 7,00,000 10,00,000
Ginger Purchase 4,00,000 7,00,000 9,00,000

The company has no previous debit/credit balance in its VAT return. The Company
needs your advice to get refund of the VAT amount for these months. 5

b) Horizon Enterprise, Kathmandu has wholesale transactions of clothes and rice from
two business houses with same PAN. It has the following transactions for 2073
Bhadra to Kartik:
Months Clothes Sales Rs. Rice Sales Rs.
Bhadra 2073 1,00,000 3,00,000
Aswin 2073 5,00,000 5,00,000
Kartik 2073 5,00,000 6,00,000

The enterprise recorded its clothes sales Rs. 31 lakh and rice sales Rs. 20 lakh from
Bhadra 2073 to Shrawan 2074. Then, it has following sales in 2074 Bhadra to Kartik:

Months Clothes Sales Rs. Rice Sales Rs.


Bhadra 2074 10,00,000 3,00,000
Aswin 2074 16,00,000 5,00,000
Kartik 2074 10,00,000 6,00,000

Tax officer has issued a notice to register the business in VAT on 2074/05/10 to the
enterprise. The tax officer has calculated interest, additional fees, and penalties as per
VAT Act. Justify whether the enterprise is required to register in VAT or not. If it is
required to register, then when it has to do so and from when it has to collect tax? 5
Answers:

a)

Section 24 of VAT Act, 2052 has provisions for VAT refund. As per the provisions, when tax
becomes receivable for the month, excess amount may be adjusted with any outstanding amount, if
there is still receivable amount, remaining amount may be adjusted with the payable amount for the
next month. If it is continuous for 6 months, then registered person can apply for a lump sum refund.
For the export business for the fiscal year 2073/74 and 2074/75, any registered person whose export
sales for a month are 40 % or more of his/her total sales for that month, S/he shall be entitled to a
refund of the remaining excess after adjusting any outstanding amount.

Calculation of local sales and Export percentage

Months Local sales Rs. Export Rs. Total Rs.


Jestha 2074
Garments 700,000 3,00,000 10,00,000
Ginger 4,00,000 1,00,000 5,00,000
Total 11,00,000 4,00,000 15,00,000
Percentage 73.33 26.67 100
Ashad 2074
Garments 6,00,000 6,00,000 12,00,000
Ginger 5,00,000 3,00,000 8,00,000
Total 11,00,000 9,00,000 20,00,000
Percentage 55 45 100
Sharawan 2074
Garments 5,00,000 10,00,000 15,00,000
Ginger 6,00,000 4,00,000 10,00,000
Total 11,00,000 14,00,000 25,00,000
Percentage 44 56 100

Calculation of output tax and input tax credit


As per schedule 1 of VAT Act, ginger is vat exempted items, only garment is VAT attractive item
products. Export has 0 rate vat. So, VAT output is applicable only local sales garments and input is
applicable only for the purchase of garments.

Months Local sales VAT Purchase Rs. VAT Paid Rs. VAT
Rs. collected Receivable
Rs. (Rs.)
Jestha 2074 700,000 91,000 8,00,000 1,04,000 13,000
Ashad 2074 6,00,000 78,000 7,00,000 91,000 13,000
Sharawan 5,00,000 65,000 10,00,000 1,30,000 65,000
2074

As per the above mentioned provisions and export percentage, the silver export and import can make
adjustment of Rs. 13,000 for the month of Jestha, 2073 continuously next 6 months, if it is not
adjusted, then after it can claim for refund. It has less than the 40 % export during the month.

For the month of Ashadh and Shrawan, it has export more than 40 percent, so it is entitled for refund
of those receivable amounts during the filing of VAT Return.

b)

As per Rule 6 and Rule 7, there is exemption from registration to small vendors having 50 lakh or less
taxable transaction of goods during the previous twelve months. If the turnover of transaction carried
on by any person exceeds 50 lakh, the person carrying on such transaction has to make an application
in the format referred to in Schedule-1 to the concerned Tax Officer to get the transaction registered
within 30 days of the date of such excess.
Determination of Turnover Threshold- Rice is nontaxable sales, as such; the turnover of rice does not
have any role in determination of threshold for registration
Total Taxable Sales in last 12
Month
months (in lakhs)
12 Months to Shrawan 31
2074
12 months to Bhadra 2074 31-1+10= 40
12 months to Ashwin 2074 31-1-5+10+16= 51
12 Months to Kartik 2074 31-1-5-5+10+16+10=61

The Enterprise has tax exempted and taxable transactions, the taxable transaction exceeds 50 lakhs
(31 lakh-1 lakh-5 lakh + 10 lakh +16 lakh) on Aswin, 2074, the actual date is not given in the
questions, and only 1 lakhs is exceeded than prescribed limit.

The notice issued by tax officer is not compliance with VAT Act and Rules, total transactions of Rs.
51 lakh (31+20) from Bhadra 2073 to Sharawan 2074 is including VAT exempted goods.

The Enterprise is required to submit application within 30 days from Aswin, 2074; it can collect tax
after registration on VAT.

8. Write short note: (4×2.5=10)


a) Circumstances beyond Control
b) Purchase of Under Invoiced Goods
c) Taxable Value for Imported Goods
d) Tax Period
Answers:
a) Circumstances beyond control as per Value Added Tax Act, 2052
VAT should be deposited within due date. If a tax payer fails to deposit VAT within due date, then tax
payer is subject to fine at the prescribed rate. However, tax payer can apply for waiver of such fine to
the Department, and Department may waive such fine if the department is convinced that the tax
payer has not deposited the tax due to circumstances beyond control. Rule 35 of Value Added Tax
Rules 2053 specifies below cases as a situation beyond control:-
1. In case the person required to pay tax becomes disabled due to falling ill; up to seven days of the
date of his recovery.
2. In case the person required to pay tax is to observe obsequies; up to seven days of the end of the
obsequies,
3. In case a woman required to pay tax delivers a child; up to thirty five days of the date of delivery,
4. In case the person required to pay tax dies or becomes insane or disappears and his heir or
guardian submits an application within thirty five days of the date of such incident; up to seven
days of receipt of such application,
5. In circumstances when the person required to pay tax has not been able to come to the Tax Office
because of the closure of a road due to floods, landslides or similar other reasons; up to seven
days of opening of the road,
6. In circumstances when he cannot come to the tax office due to total haltage of transport; up to the
next day of the end of such haltage.

b) Purchase of Under Invoiced Goods as per Value Added Tax Act, 2052
Sale of goods at a price lower than fair market price of such goods at the time of sale of such goods is
known as sale of goods at under invoice. VAT is levied on a transaction value which is a fair market
value. Hence, if any tax payer shows sale of goods at a price lower than fair market price, then there is
avoidance of VAT, and loss of revenue. To protect such activity, VAT Act, 2052 has provided
specific provisions under section 23ga of VAT Act 2052 which provides special rights to Tax officer
to sale or purchase such under invoiced goods. As per the provisions of section 23ga,

1. Notwithstanding anything mentioned in any other laws in force, if any person is found to have
shown transaction by under invoicing by selling goods at a price less than the fair market price,
then Tax Officer can stop the further sale of such goods which are in stock, and may sale or order
for sale such goods at the under invoiced price.

2. If the person refuses to sale goods while purchasing or causing to purchase by the department or
Inland Revenue Office as per sub/rule (1), the department or the office may take in possession of
the goods and shall pay the amount, calculated on the basis of under invoice value, to the person
when the person comes to receive the payment.

3. The goods purchased or caused to be purchased pursuant to Sub-sections (1) and (2) may be sold
or caused to be sold at such price and in accordance with such procedures as may be specified by
the Director General.

c) Taxable value for imported goods: According to Sec. 12(5) and Rule 48; in case of goods, respective
custom office collects VAT based on taxable value. Section 12(6), provides the revaluation of
purchase price at market price in case of significant under-invoicing found or reasons to be believed.
Taxable value for the purpose of import shall be computed as follow:
Value paid to vendor or value as determined for custom duty
 Custom duty and custom service charge
 Excise or countervailing duty
 Other duties
 Demurrage or other penalties
 Insurance and transportation
d) Generally, it means period for which VAT assessment is carried out. Tax period for VAT is inter-related
to the related VAT Return.
Tax Period means a period prescribed by the Act or Rules for calculation of net VAT payable or
receivable.
A registered person has to adopt a month as per Nepali Calendar as tax period. The tax starts from day1 st
of a month and ends at the end day of the same month.
As per the provisions (Rule 26) of VAT Rule, if deemed fit by the tax officer, different tax periods may
have to be adopted / accepted by certain specified tax payers.
i) Tax Period of 2 months:
Under such circumstances, the tax period may be Shrawan and Bhadra, Ashwin and Kartik, Marga
and Poush, Magh and Falgun, Chaitra and Baisakh, and Jestha and Ashad. This tax period is generally
adopted by hotel and tourism enterprises if specifically applied for the same.
ii) Tax Period of 4 months:
Tax period of 4 months is adopted by tax payers specifically, if paper or electronic media or brick
producer applies so.
iii) Different tax period:
In case the registered person maintains its accounts in the approved computer systems, on application
by the tax payer, the tax officer can allow different tax period to be adopted provided the tax officer
deemed fit.
iv) Tax period for 1st time of registration:
In such cases, tax period starts right from time of its registration to the end of the month.
- End-

You might also like